September 16, 2011

Mitch explains SAT trends

Respected commenter Mitch explains what is going on with rising Asian SAT scores:
It's a mix of things. 
First, the 2005 changes made the math subject material easier, but an actual high score became harder because there were fewer hard problems and the impact of "unforced errors" became higher. Kids who were bright but careless could get a high score because the occasional unforced error was wiped out and more with their performance on the high difficulty questions. That last sentence describes whites more than Asians.  
So since 2005, the ability to nail every question and not make unforced errors--something that drill does, indeed, help--has been rewarded, whereas the number of creatively difficult problems is 0 or 1 per test. This hurts white students, on average, more than Asian students just by personality trait, and then the Asian tendency to drill for this test gives even more of an advantage.  
The reading test has been made unequivocally easier. I'm not sure what you mean about some reading questions being moved to the writing section. This is not true. The writing test is a near-exact replica of the old English Composition Achievement test, or the English Writing Subject test. There were no changes to it at all from a content perspective--they just changed the type of essay prompt and reduced the number of questions. 
Certainly, the easier reading test makes it easier for Asians to get high scores. The writing test rewards attention to detail above all 
So the test changes play a part, both in how they reward the traits more likely to be in asians over whites, and then in the Asian prep ritual--which really has to be seen to be believed. I teach in these schools, and the kids are in prep taking tests for 2 years. Even the ones who aren't getting super high scores are getting better scores, and that's bumping the average up. 
Then there's the fact that Koreans, Chinese, and Indians are immigrating here in huge numbers, which is presumably offsetting the once larger percentage of Filipinos and Tongans.  
I'm assuming you were only looking at US students, right? [I don't know -- it's not obvious from the College Board documents.] Koreans in Korea are taking the test as well, and there they study 40-60 hours a week, instead of school. The prep schools there buy copies of the most recent tests from students and use the tests to prep their students (something that's frowned upon here, although not technically illegal). The kids learn how to write essays by rote, and have whole essays memorized (use this essay for "change", this one for "education", and so on).

There's a general lesson here. Our society is constantly changing and bending procedures in hopes of Closing the Gap between the more feckless and the more feckful racial groups. Yet, perhaps unsurprisingly, the more feckful tend to better exploit these changes for their own benefit. 

For example, the changes in federal regulator attitudes toward zero-down and low-doc mortgages that George W. Bush announced in 2002 in the name of fighting racist redlining at his White House Conference on Increasing Minority Homeownership  poured hundreds of millions of dollars into Angelo Mozilo's net worth, most of which the SEC allowed him to keep even after fining him. But the net worth of the median Hispanic and black is now lower after mortgage follies.

The changes in the SAT in the last decade were mostly due to a single man, University of California chancellor Richard C. Atkinson. The UC system is the College Board's biggest client, and the UC was told by California voters in 1996 that racial preferences were now a violation of the state constitution.

Atkinson held an inflated notion of his expertise in the field of psychometrics. "When students asked me about IQ testing, I frequently referred them to Stephen Jay Gould’s book The Mismeasure of Man, published in 1981; it is a remarkable piece of scholarship that documented the widespread misuse of IQ tests," he wrote in an essay explaining part of his motivations, although he disingenuously left out all mention of the 800 pound gorilla in the room for UC, Proposition 209. The state legislature's Latino Caucus had threatened to cut the university system's budget unless they could figure out a way to cheat on Prop. 209 and get more Latinos admitted.

Now, Atkinson wasn't a complete cynic. He was more of a fool. He thought he could kill a whole bunch of birds with one stone. For example, he wanted the SAT to be more of an achievement test than an aptitude test because of his quasi-Gouldian views on IQ testing:
My views are similar to those of Alfred Binet, the French psychologist who, in the early years of the last century, devised the first IQ tests.  Binet was very clear that these tests could be useful in a clinical setting, but rejected the idea that they provided a meaningful measure of mental ability that could be used to rank order individuals.  Unfortunately, his perspective was soon forgotten as the IQ testing industry burst onto the American scene. 

This is like saying:
My views are similar to those of Nicolaus Copernicus, the Polish astronomer who, in the middle of the 16th Century, devised the first heliocentric system. Copernicus was very clear that heliocentrism reduced the number of crystalline spheres and epicycles necessary, but rejected the idea that we could explain the motions of the planets without the Music of the Spheres. Unfortunately, his perspective was soon forgotten as Kepler, Newton, and Einstein burst onto the astronomical scene.

Atkinson's goal was to make the SAT less of an aptitude test and more like the SAT II Subject Tests, which are more achievement tests. (UC long demanded applicants take both the SAT and three SAT Subject Tests, a high degree of overkill.) Since we've all know about how blacks and Hispanics are obviously oppressed by the white man's IQ pseudo-test, the thinking went, this would ameliorate UC's little problem with the Latino Caucus by Closing the Gap.

Except, it didn't work out that way because blacks and Latinos tend to not only have less aptitude, but they tend to be lazier about achieving academically. Also, it made the new three-part SAT 3.75 hours long, which may tend to mentally exhaust blacks and Latinos. The winners from these "reforms" turned out to be Asians, who, on average, work hardest. 

Of course, the UC schools already had plenty of Asians. So, a few years ago, UC, having turned the SAT into something like the SAT Subject Tests, announced it wanted to stop requiring the SAT Subject Tests as now being redundant. This sensible reform, however, outraged the Asian-Pacific Islander caucus in the California legislature. The more tests parents have to remember to sign their kids up for and pay for tutoring in how to beat the tests, the better Asians do versus everybody else.

One thing to keep in mind is that elite private colleges don't really seem to want more Asians (at least if they're not going to pay list price. Foreign Asians, who don't qualify for financial assistance, are increasingly fashionable with colleges.). The more Asians score high on the SAT and get inflated GPAs by taking a lot of AP courses, the more the super-prestigious private colleges, which can and do use quotas for admitting blacks and Latinos, appear to discriminate against Asians.

Why they discriminate against Asians has been speculated about, but not, so far as I know, studied in any truly illuminating fashion.

Do they not donate as much money to their alma maters? My vague impression is that Indians, with their ancient tradition of alms-giving, are pretty generous when they have a chance to get their name put on an academic building. (I recently took enthusiastic part in a three minute standing ovation for an Indian gentleman who was the chief donor for the new library at my son's high school. It's a really nice library and it was all paid for and build by the week before my son started school there, so it didn't cost me a penny, so clapping my hands sore was the least I could do.) Chinese benefactors? Maybe not so much ... I don't know. This is the kind of thing that colleges have no doubt studied in intense detail, but their findings are Top Secret.

Or maybe too many Asians is considered uncool by high school students. For example, UC Irvine has long been heavily Asian, but it never seems to climb in coolness with kids. 

Or maybe the elite colleges just don't believe the high SAT scores being recently recorded by Asians. Who knows? Obviously, elite private colleges know what their motivations are for requiring higher paper credential from Asians, but they aren't telling, and nobody seems to be asking.

185 comments:

Anonymous said...

What's a creatively difficult question and how did Mitch conclude that there were only 0-1 "creatively difficult" questions per test?

Anonymous said...

I'm looking at your previous chart Steve and one thing that I noticed was that from say 2000-2005 the Asian improvement in SD units relative to whites on the math and verbal sections of the SAT was about the same as the improvement from 2005-2010. This seems to cast some doubt on Mitch's theory that the new SAT test in 2006 significantly favored Asian Americans relative to whites because they somehow got rid of certain question types.

Anonymous said...

I thought the SAT math was made harder in the new 2006 test? They added some higher level algebra 2 material, amongst other things, if I recall correctly.

Anyway, making the SAT math test harder is a pretty bad way of closing the Asian-white gap. Just look at a really tough math test, like the AIME. The Asian-white gap is pretty significant on that, considering that in recently years something like 55-60% of USAMO qualifiers are of East Asian descent.

Anonymous said...

Maybe the reason Koreans are less likely to donate a few million dollars to have their name on a building is because they all have the same name?

nooffensebut said...

This seems like a good opportunity to remind everyone that Ivy League undergraduate Affirmative Action comes at the expense of Asian-Americans 80% of the time and only 20% at the expense of white applicants, per Espenshade and Chung. I have considered this one of the primary vulnerabilities of Affirmative Action rationale. Why hurt Asians? Asians weren’t discriminated against? Asians aren’t diverse? Now, Fisher v. Texas and the latest wrangling over Michigan’s Proposal 2 could make Affirmative Action a campaign issue again and force Supreme Court intervention.

Anonymous said...

Why do top schools have Asian quotas? Because between higher Asian SAT/GPAs, and affirmative action for NAMs, people would notice the complete lack of white kids, and someone might ask Unfortunate Questions.

--Discordiax

L said...

Nice to finally see some sensible acknowledgement in the HBD sphere of what has always been fairly obvious to any objective onlooker.

Asian performance on tests has much to do with hard work and less to do with intelligence, which makes sense considering lukewarm Asian intellectual performance for the past 150 years.

That, by the way, is the 800 pound gorilla in the room - considering there are so many more Asians than whites, and they have higher scores, why do they so consistently fail to equal whites in intellectual performance?

One hopes that this kind of sensible analysis will begin to be applied to IQ tests as well.

I think its fairly obvious why elite schools dont want more Asians - high Asian scores dont seem to lead to the kind of elite intellectual performance one would expect, and the schools know this from experience. So elite colleges risk courting intellectual mediocrity by becoming too Asian and they wish to avoid that, quite sensibly.

Asians have really thrown a wrench into the system here, in that they get great scores which somehow dont lead to elite inellectual performance. This should make us re-evaulate our tests and develop better ones, because the ones we currently have are clearly failing - they fail to predict what they are supposed to predict, elite intellectual performance.

I think its quite obvious our tests dont measure what we used to think they did, and Asians have shown that to us.

Anonymous said...

Is Mitch seriously complaining about Koreans in Korea "gaming" the SAT? These people aren't even native English speakers for Christ's sake! Imagine if white Americans had to take some kind of entrance exam in Chinese...

Mitch said...

Anonymous 2:34:

The SAT added a bit of second year algebra onto the SAT in 2005, which supposedly made it harder--except it didn't, not really. The only thing kids had to know that they didn't have to know before was function notation (f(x)), whereas the SAT before just used symbol problems.

But what is never mentioned is that all sections of the SAT became shorter, because when they added in the writing section, the test became huge. To keep it at 3:30, they had to cut questions. The Reading test went from 78 (I think) to 66, the math section from 60 to 54, and the Writing (which had been the English subject test before) went from 60 to 49.

But if they have fewer questions, something has to give. They couldn't eliminate easy questions from the test, because that would skew the achievement gap even worse. I have the original red copy of the book Real SATs, as well as the blue 8 sample SATS book they put out after the changes. There is simply no comparison.

They hid this in the books, by the way, by changing the way they rated problem difficulty. Originally, they ranked problems from 1-5. For a number of years after the change, they ranked the difficulty 1-3. (They may have changed that, but I'm not sure. It is recent, if so.) So you couldn't do a like to like comparison and check percentages. I looked for similar problems, though, and could find very few 5 (high difficulty) questions in the blue book. Most of the 3 (new high difficulty) questions I could find were ranked 4 in the old book. But it's not a perfect comparison.

Steve, I'm pretty sure donations and desirability play a big part in the discrimination against Asians. At the local level here, we have lots of traditionally white high school areas that are being taken over by the cocooning process (Asians find it, think it's a good school, and move there in large numbers). Schools in wealthy districts are fighting it tooth and nail (by coming up with policies that upset Asians, like ending student rankings and limiting AP classes), and the only reason that makes sense is that they fear Asians will give less money. (And they are much higher maintenance parents; anecdotally, I know of several teachers who left highpaying Cupertino schools because the parent pressure was absurd. I dunno, though.)

Oh, and thanks for the "respected"! I am much honored.

Anthony said...

Or maybe the elite colleges just don't believe the high SAT scores being recently recorded by Asians. Who knows?

I suspect that it's some of this, though probably expressed as a preference for "creativity". Nobody has a good way of measuring people's creative potential, and people's creative achievement (especially their creative achievement before age 18) is tremendously subject to outside influences. But all else being equal, Asians do seem to be less creative than whites.

Colleges may also believe some of their hokum about "preparing tomorrow's leaders", and want to find smart kids who show "leadership potential" - something which is underrepresented in Asians (and Hispanics) but overrepresented in Blacks.

Sheila said...

They did this with the standard IQ tests, too - removed the higher- end and most difficult questions, so the highest possible IQ score is now significantly lower and does not accurately reflect the abilities of the extremely gifted. The old Stanford-Binet went up to 200; the WISC stops around 160 now, I believe.

Test taking, colleges - it's all become an immigrant racket and I'm keeping my kids out of it. Private school and then community college/work-study/online courses. I will not go into debt to have them "indoctrinated" and they don't need a devalued BA to get minimum wage jobs. White Americans need creative alternatives to avoid the Jewish/Asian "elite" colleges competition racket.

Yan Shen, go home.

Yan Shen said...

Sheila, America is my home. I love the United States more than anything else in the world.

Felix said...

Elite colleges don't "discriminate" against Asians, they simply give a boost to NAMs and the children of the rich and powerful. There is only one reason that the "white" score for admits at elite colleges like HYPS is lower than the Asian score.

The reason is, simply, that a lot of the white kids that get admitted to those schools are the children of senators, large benefactors, corporate CEO's, professors, and other members of the elite. Since the status of the parent does not always correlate with the intelligence of the children, a lot of those kids would never have been accepted were it not for their parents. On the other hand, there aren't nearly as many powerful/rich Asians so Asian kids who get in do so on merit.

Do not take that to mean I'm suggesting some kind of 'white privilege' here, far from it. If you're a white male whose parents are nobodies, good luck trying to get accepted to Harvard with SAT scores that are at the "white" mean for admits. Good luck even with an Asian mean. The truth is, if you take a white kid and an Asian kid trying to get into Harvard and you normalize for parental status, the white kid if anything will need a higher score than the Asian kid to get in.

Let me put it more simply: HYPS accepts 15,000 student a year total (just making it up), of which 8,000 are white and 4000 are Asian. Out of the 8,000 whites accepted, 2000 are because their parents are large donors, former presidents, current senators, Fortune 500 CEO's, etc. The SAT scores of those 2000 kids play very little role in their admissions, cause who cares about some stupid number when your dad is a US senator. On the other hand, of the 4000 Asians accepted, maybe 20 have parents that are big deals and the rest get in because of high scores. So it's easy to see that if you have 8,000 whites getting accepted, of which 25% are accepted for reasons other than SAT scores while only <1% of Asians are accepted for such reasons, then even if the great majority of whites who are accepted have higher SATs than Asians, the Asian mean will still be higher because of that 25%. There are plenty of white kids who are NOT getting accepted to HYPS that have SAT scores much, much higher than those kids of the elite. If you replaced "elite" kids with those kids you'd see the white mean climb like a rocket.

Marco Lalo said...

So test-prep and studying is now bad?

You guys are turning values upside down all in the service of... I dunno.

Anonymous said...

Gotta disagree with you here completely Mitch. I can see whites benefiting from a "harder" reading section on the SAT, but I find it extremely hard to believe that making the SAT math test harder, along the lines of say the AMC or AIME exams, would actually close the Asian-white gap.

Mitch said...

Oh, I knew there was something I wanted to mention:

You are generally correct about the subject tests, but there's a few things to add:

Saul Geiser's research about the predictiveness of the SAT led to Atkins decision. The research suggested that the subject tests were a better indicator of FYGPA than the SAT. However, I've always wondered if Atkins misread the research and then went back to Geiser for more guidance. While he originally signaled that he was going to eliminate the SAT requirement, Atkins ultimately doubled down on the SAT.

Remember, while 3 SAT subject tests were required, students only had a choice on one of the three tests. One test was the English test (now the writing portion of the SAT), and one test was the Math 1C, a now mostly defunct test, since the UC no longer requires it. The English and Math 1c tests were MUCH easier than all the other subject tests. This can't be overemphasized. When people say "research shows the subject tests were more predictive than the SAT", this is--well, for starters, it's inaccurate, as that's not what the research showed. But more importantly, what the research *did* show about the subject tests involved the two tests that were, by far, the easiest of the subject tests.

So the original talk was of the UC relying only on the Subject tests. Then, suddenly, all that changed:

1) The Writing test (again, a much easier subject tests) was rolled into the SAT.

2) The Math 1C test (which was not as easy as the writing test, but much easier than the other tests) was eliminated as a requirement. The SAT I math test subject matter content was adjusted to include some second year algebra, but it was not in any way a replacement. For example, the Math 1c includes trig, while the SAT was not.

3) Students now took two subject tests of their choice.

So consider: what the UC actually did, after all the talk, was increase the weight of the SAT, and nearly halve the math content and difficulty of the required tests.

And from that point, they then devalued the subject tests--which used to be twice the weight of the SAT--and ultimately eliminated them.

Why? First, you're right--the achievement gap was worse. But more importantly, why did UC never go to all subject tests, even though it had originally planned to?

My guess: they realized how many false negatives they'd get. A kid who can read and write well enough to score a 600 or higher on the SAT will not do well on the English Lit or US History test if he didn't care much about those classes. The SAT, (and its cousin, the ACT) for all the complaints, is an exceptionally egalitarian test. It fairly indicates whether or not a student is ready for college. In fact, if we changed the question from "does the SAT/ACT predict college success better than the subject tests or GPA" to "does the SAT/ACT predict college *readiness* better than the subject tests or GPA", we could all go home, because that question has already been answered. Just go look to see how many colleges use SAT/ACT scores as "get out of remediation testing" proxies.

So while the press was always reporting about the UC threatening to stop using the SAT, I suspect the reality was that the UC had already figured out they couldn't do that--first, because they'd get too many false negatives and second, because any real reliance on the subject tests would exacerbate the achievement gap.

Anonymous said...

To go off of what I previously stated, just look at the over-representation of East Asian Americans amongst say USAMO qualifiers. In recent years, East Asians alone have made up 55%+ of USAMO qualifiers.

So we have empirical evidence directly contradicting what Mitch is claiming here, that somehow making the SAT math test harder would close the Asian-white gap. In fact, the East Asian American over-representation amongst USAMO qualifiers, which is determined by your performance on the AMC and the AIME exams, both of which are significantly harder than the SAT math, is higher than say amongst 800 SAT math scorers. Bottom line, on mathematics exams, increasing the difficulty increases the Asian-white gap rather than reduces it.

Anonymous said...

How long will it take for us to get the detailed demographics release for pisa data for math to compare to this?

Anonymous said...

Ok.So why should Asians donate to institutions that actively discriminate against them?

For that matter why should non-elite white people support liberal colleges that actively work to undermine white people?

This whole donating to your college is all about a way to justify discriminating against asians.

Anonymous said...

This is hilarious. All the effort to prove that black people cannot catch up to whites no matter how hard they try in HBD circles comes unraveled when it comes to Asians.

Apparently the reason that blacks dominate the NFl combine is because whites and especially Asians do not spend enough time in the gym.

Kaz said...

@Shelia

Seriously stop posting, you're a horribly misinformed parent if you think CC is a better alternative than your flagship state uni. Why send your kids to a CC when they can go to a state school that is probably highly ranked, cheap, and will get you a job with the large companies in the area after graduation? Also online courses? Those are a scam and don't prepare you for nuts. There is no reason to get into debt pursuing a BA/BS only idiots who go to mediocre private schools do that.

Anonymous said...

In fact, UC Irvine is known as the University of Chinese Immigrants among Chinese.

Kudzu Bob said...

I love the United States more than anything else in the world.

That's a lie. If you loved the United States, you would not want to see it altered beyond recognition, like something out of Bladerunner.

Hacienda said...

"Also online courses? Those are a scam and don't prepare you for nuts."

Don't tell her that. If she really believes Caltech is a scam and Kaplan Online University is the real deal, then she deserves what she gets- nothing.

David Davenport said...

Foreign Asians, who don't qualify for financial assistance, are increasingly fashionable with colleges.)

That's an important reason why American U.'s push multi-kulti diversity:

The U.'s want to attract more foreign students, including Muslims, provided that either the students or the US taxpayer pay full, list price tuition.

The foreigners don't qualify for financial assistance at present? The American higher ed. mafia is lobbying to change that.

Mitch said...

Is it possible that people don't understand what I'm saying? I am not saying that Asians would do worse if the test were harder. There is a top limit, though, so there's only so high they can go. I am saying that making the test shorter--thus increasing the impact of each missed question--and easier (the quant analysis alone reduced the abstract thinking skills quite a bit)--a certain category of student will get lower scores (say, 650-720 instead of 720-750), and that this category of students is more likely to be white.

Dennis Dale said...

I had to break off reading just to say: the Binet/Copernicus analogy is effin lovely.

Anonymous said...

I'll defend Sheila.

Don't bother sending your kids to Highschool.

Send them to Community College instead.

Highschool is a waste, SAT a waste....start them with some simple classes around 4th grade in the CC and then by the time they are 9th grade they are done with Vector Calc.

Problem. Solved.

Asians. Rerouted.

SAT Test meaningless.

Worldly Wayward Woman said...

So since 2005, the ability to nail every question and not make unforced errors--something that drill does, indeed, help--has been rewarded

That's interesting. In other words, the College Board inadvertently began tested for conscientiousness.

There's only one way to handle this equitably-- the same way Selective Service drafts soldiers-- by lottery. Set a minimum score for each school (say, the lowest SAT score in last year's entering class) and every college senior who applies with an SAT above that line gets a ticket. If nothing else, it would make high school a lot less stressful. And it'd be hilarious to see how enormous an applicant pool the SAT scores of scholarship athletes would create ("lowest score in last year's entering class").
Oh yeah, and bring the Selective Service draft back to at the same time to redirect some of the rage of affluent mothers.

Dominion of Canada said...

To go off of what I previously stated, just look at the over-representation of East Asian Americans amongst say USAMO qualifiers. In recent years, East Asians alone have made up 55%+ of USAMO qualifiers.

I don't want to play the excuses game, but I had a thought on this. This could just be another instance of tiger mothering.

Some of the smartest white kids I know had parents who left them alone, and they spent their teen years having fun building circuit boards and pipe bombs in the garage, and playing Magic: the gathering instead of trying out for the math olympiad.

Anonymous said...

Asians do just fine in life when they're out of college. They just don't do as well as their higher IQ scores and strong academic performance would lead you think.

Jews, to pick another high achieving group, do outstandingly well in real life.

If a college has to pick between a hardworking Asian kid who will become a competent physician/engineer/dentist or a quirky Jewish kids who could invent Facebook II, they'll probably go with Zuckerberg. Asians are useful in that they bring as serious focus to the campus, but they're perceived as boring too. So some, but not too many, is the ideal.

Recruiting blacks and Latinos over smarter Asians makes sense too, as blacks and Latinos are likely to go on to success because of AA. Harvard likely admitted Obama under AA, considering Obama's mediocre grades, but it turned out to be a good investment. If they had gone with Amy Chua instead, their investment would've produced much lower returns.

Anonymous said...

This trend of Asians taking over good school districts is going in a lot of places outside of California. In Seattle, there's a trend of high income Asians moving out to the affluent eastside suburbs because of the reputation for safety and good schools.

Anonymous said...

I'm asian and I won't ever donate a penny to any of my alma maters because I was always treated as an uncool outsider.

RKU said...

I never cease to be amazed how HBD-people can dispute, seemingly with a straight face, the obvious fact that Asians are (on average!) quite a bit smarter than whites. After all, on one side we have a huge mountain of evidence, while on the other we merely have some noisy hand-waving. In fact, probably the strongest evidence against the hypothesis would be the vast amount of research by prominent university academics claiming that all ethnic groups have exactly the same innate characteristics, differing only in their social and environmental advantages; so perhaps some of the commenters here should begin citing Gould and Lewontin...

Now admittedly, the Asian/white gap isn't enormous---certainly much less than a single SD. So perhaps the commenters are really meaning to say that Asians aren't any smarter than *smart* whites, which is certainly correct. In fact, I'd suspect that the Asian population wouldn't seem too different than the white one, if you just excluded the dimmest one-third of whites, who anyway aren't likely to read this blogsite---or much of anything else for that matter. And I wouldn't be too surprised if a few of the harder-core HBD-people would secretly desire to do exactly that and "exclude" those dim whites, so perhaps their argument contains an element of wishful thinking.

Also, it's certainly true that for cultural and quite possibly genetic factors, Asians tend to be less aggressive and more socially-conformist than whites, perhaps with fewer psychotic tendencies as well. On the plus side, this makes Asian countries and communities much more quiet and orderly, but it's also perfectly reasonable to suspect that these same factors might tend to somewhat diminish Asian achievements or breakthroughs at the very high end, given that "genius" has commonly been linked with "madness" or at least the willingness to flout established norms.

On a related matter, I'd suspect that at least part of the growth of Asian crammishness is due to a vicious academic cycle. The Asian population has rapidly grown and also become more affluent, while elite universities have mostly imposed crypto-quotas, putting an effective cap on Asian numbers. This has intensified the Asian competition for those limited slots, leading to massive Tiger Mom-style cramming, leading to higher test scores, leading to sharper discrimination, leading to more cramming and so on. The result is a horrible scholastic arms race in which everyone---certainly including non-Asian students---generally suffers, and almost nobody seems to really benefit.

Meanwhile, the vast majority of (gentile) whites are caught in an exceptionally discriminatory pincer attack, in which sizable groups on one side of the performance curve must not be substantially under-represented while small groups on the other side of the curve may still be massively over-represented, leaving relatively little left over in the middle, especially when the unbrilliant children of CEOs, Senators, and celebrities have taken their fill.

I actually think I came up with a pretty elegant possible solution to all these problems a few years back, and perhaps one of these days I'll actually get around to writing it up...

Anonymous said...

quirky Jewish kids who could invent Facebook II, they'll probably go with Zuckerberg

Considering that Zuckerberg stole the idea from the Winklevoss twins, why don't they go with more kids like the Winklevoss twins i.e. white Christians who are greatly underrepresented at Harvard and other elite colleges as Ron Unz has shown?

http://buchanan.org/blog/pjb-the-dispossession-of-christian-americans-241

Dominion of Canada said...

I never cease to be amazed how HBD-people can dispute, seemingly with a straight face, the obvious fact that Asians are (on average!) quite a bit smarter than whites.

It just gets annoying hearing it constantly. Especially when the reality is that whites have been, and still are, responsible for the majority of the world's technological and social advancements, and create the world's most liveable countries.

So it just doesn't square with Asians having higher SAT scores, and we come up with (plausible, imo) excuses.

Anonymous said...

2Blowhards Blog:

2B: "I am living in China, a country where there is said to be a high average IQ. But interacting with Chinese people, I feel that a lot of that high IQ is just wasted, because the culture is conformist. Put another way, Chinese people seem to have lazy, intellectually uncurious minds. If you talk to Chinese people about nearly every issue, you tend to find that 99.99% of people just repeat back the Communist Party's views. You could sit a Chinese person down and ask him to memorize the dictionary, and a fair few Chinese would anally plod through, but if you ask his views on a practical question or social issue (from how to mend a broken door to questions of nationalism and democracy), the result is extremely disappointing. It seems Chinese people are intelligent but gormless, if you get the distinction. Why should such a pattern develop/evolve? Why would such a nation need a high IQ if they are culturally constrained from using it?"

http://www.2blowhards.com/archives/2009/01/a_week_with_gre_2.html

Anonymous said...

http://www.reuters.com/article/2007/09/20/us-taiwan-children-idUSTP27407920070920?feedType=RSS&feedName=lifestyleMolt

Taiwan children are getting shorter and educators believe study pressures, poor diet and too little sleep are partly to blame.

The average height of Taiwan children in fourth, fifth and sixth grades dropped by 1 cm (0.4 inch) a year between 2003 and 2005, Taiwan's Education Ministry said on Thursday.

Possible explanations are improper diet, insufficient exercise and poor sleep caused by after-school studies, educators say. Ambitious parents take children to English, arts and music lessons after elementary school classes end at 3 or 4 p.m.

"Some of us don't sleep enough or exercise enough, while some kids are a bit overweight and don't want to work out," said Chen Tzi-hsuan, 12, a Taipei sixth-grader. "And there are some children whose parents make them do extra study."

Anonymous said...

Lots of good comments. I like L, Felix, and, um, anonymous.

Here's the thing. If a school wants to test your conscientiousness, it can do that by looking at your grades. When it looks at SAT its trying to figure out pure "G", but all that cramming gets in the way. They have to adjust Asian scores down in order to get an accurate measure of "G".

At the elite level Asian conformity and boringness is seen as a detriment. I heard many a time on the street that Asians were under performing in IB. Truth is, they were probably over performers in life that were overwhelmed by punching above their belt "G" wise.

Anonymous said...

Public school students in Shanghai often remain at school until 4 p.m., watch very little television and are restricted by Chinese law from working before the age of 16.

“Very rarely do children in other countries receive academic training as intensive as our children do,” said Sun Baohong, an authority on education at the Shanghai Academy of Social Sciences. “So if the test is on math and science, there’s no doubt Chinese students will win the competition.”

Kaz said...

@Anonymous 9:31

lol if you wanna say who Zuckerberg purportedly stole the idea of Facebook from at least mention the Winklevoss' partner (i.e. co-founder of HarvardConnection/ConnectU) Divya Narendra.

But that would dilute your message wouldn't it.

Anonymous said...

The entire post sounds like sour grape.

Anonymous said...

I never cease to be amazed how HBD-people can dispute, seemingly with a straight face, the obvious fact that Asians are (on average!) quite a bit smarter than whites.

It's not really settled whether they are a bit smarter, like Northern Europeans to Southern Europeans (this isn't really so true in the US as it's mostly cultural between these two), or a lot smarter like Whites are to Hispanics or Middle Eastern people. They might have gains some study or other, less genetic, achievement factors.

I know there's this who Lynn and Rushton thing of "Oh they're Mongoloids, so they have this r-K thing and have a similar relationship to Whites as Whites do to Blacks" and this was established a long time ago in the Ice Age, but this tends to ignore the very strong similarities between East Asians and the not very academically impressive people of South East Asia like the Thais (if you read Lynn's papers, he tend to very strangely lump the Thais and Malays in with Papuans and Melanesians, while they are patently very much more similar to the Chinese - likewise Lynn does very strange things like using the steppe Asians, who don't have high IQs but do have large brains as explanations why the settled Asians like Chinese and Japanese who don't have particularly large brains, as proof that Asian intelligence comes from large brains).

Anonymous said...

I agree with a couple of the posters here who say that Asians on average are much smarter than Whites.

Anonymous said...

Sheila, America is my home. I love the United States more than anything else in the world.

Yan, I'm pretty sure you love Chinese-Americans more than anything else in the world, and America comes in a distant second. But distinct second is still not that bad I guess...

Anonymous said...

Asians tend to be less aggressive and more socially-conformist than whites, perhaps with fewer psychotic tendencies as well.

Asians, I think, have similar levels of empathy to Whites, but higher levels of dutiful behaviour, and lower levels of extraversion (on the whole). This is probably the big distinction. Levels of self driven desires for achievement are probably also similar between the two groups

I think dutiful behaviour is a good thing, since it's the Conservative virtue, but it's important to consider this, or else you won't really understand the character of Asian civilization, which is more moralistic and prosocial without being particularly more or less empathic or compassionate or driven by people wanting to have much more contact with people.

ZZ said...

The informal quota system used at elite private schools is a decent compromise between two extremes (excluding foreign HS students for simplicity):

SAT based admissions: 65% white, 34% asian, 1% NAM

Population based admissions: 65% white, 32% NAM, 3% asian

Actual "soft quota compromise" numbers at HYPS: 65% white, 20% asian, 15% NAM.

Shanghai Ty said...

@RKU

"I never cease to be amazed how HBD-people can dispute, seemingly with a straight face, the obvious fact that Asians are (on average!) quite a bit smarter than whites. After all, on one side we have a huge mountain of evidence, while on the other we merely have some noisy hand-waving."

Actually the huge mountain of evidence favors superior white intelligence: It's the historical record. Whites have produced wildly disproportionate intellectual contributions vs. Asians in almost every field of human endeavor over the past, what, 2000 years. The observed reality of White intellectual preeminence over centuries is a lot more than "noisy hand-waving."

Given that stark disparity, it makes sense to wonder if IQ tests are missing something important.

I realize you've hedged a bit by saying "on average." But if IQ tests seem to overstate Asian intelligence at the elite level, why do you think it's any more accurate at the mean?

Anonymous said...

Actual "soft quota compromise" numbers at HYPS: 65% white, 20% asian, 15% NAM.

Ron Unz's research showed that 20 percent of HYPS is Asian, and 25 percent to 33 percent is Jewish, Asians and Jews make up about 5 percent of the US population. So 50 percent of HYPS is drawn from about 5 percent of the US population.

The white Christian is extremely underrepresented. I don't see how this constitutes a "decent compromise".

Anonymous said...

The really terrible thing is the way that education (supposedly one of the highest of ideals), has been prostituted this way as a nasty little racial rat race, with tribal self-interests squabbling over what's 'rightfully theirs'.
I don't know what the answer is.If these attitudes persist they bode disaster for America.
Perhaps the only real answer is a real free-market in scholastic entry, where admission is openly and transparently won by test score and nothing else.

Chief Seattle said...

Whether or not Asians are smarter, it's hard to argue that on average while students are less motivated to do well, and not pushed as much by their parents.

Consider this generation the Asian version of the Scottish enlightenment. Some good will come of it. Least of all that the children of the last generation of strivers will get their comeupance.

Johnny said...

Professor Flynn has examined 14 IQ studies of the children of non-selected, pre-WWII Asian immigrants from Japan and China. These studies include the Coleman Report, Project TALENT, and the Ethnic Minorities Report. Flynn found that Asians consistently scored 0.1 standard deviation below the white mean.

At the same time these IQ studies were administered, Asians were surpassing whites on academic measures (high school graduation rate, college degrees, SAT scores) and economic measures (income, representation in high paying professional occupations). Doing further research he came to two conclusions.

First, Asians had a lower IQ threshold for accomplishing tasks such as graduating high school or entering a professional field. Second, Asians above the threshold better utilized their intellectual talents toward attaining economic and academic success.

Flynn speculated that Asian economic efficiency might be a result of the rice farmer lifestyle of their ancestors. He believes the austerity and intense work pace demanded in rice farming, had the effect of culturally and even biologically molding Asians. He contrasted this with the more lax lifestyles of European peasants.

Interestingly, in China, people from the southeastern coast (Zhejiang, Fujian, coastal Canton) are considered to be the hardest workers in the country. They've also historically formed the majority of China's successful imperial exam takers, and dominated internal and international commerce. Maybe not surprisingly, the south of China was into rice farming, while the north pursued wheat farming. The southeast seems to have had especially fertile land for rice farming, and especially intense working conditions.

If you want to understand Asians, Flynn's writings are a good place to start.

Continued below........

Anonymous said...

Continued from above......

You can also read some of the past writings about East Asians from our once politically incorrect public intellectuals. The general consensus, from liberals like Orwell to white supremacists like Lothrop Stoddard, has always been that East Asians (Chinese most of all) are highly industrious and controlled people. Even opponents of the Chinese called for their exclusion on the basis of their intense work and saving habits, which white laborers couldn't compete with. President Woodrow Wilson, who talked like a Stormfront member, even concurred that Asians should be excluded because they were "coolies."

Also, while whites were viewed as more capable of building progressive civilization than East Asians, there was a willigness to concede that East Asians were quick learners. Lothrop Stoddard had a special fear of the the talents of the Japanese, who shocked the world by defeating the Russian military.

The idea of highly industrious, moderately bright, less creative East Asians is nothing new.

Anonymous said...

Steve,

You *enthusiastically* clapped...

For *three minutes*...

Over a *library*...

Interesting...

I agree with a couple of the posters here who say that Asians on average are much smarter than Whites.

I think the most pertinent fact about Asians is that they tend to be a whole lot more asiatic than whites. Speaking for myself here, that tends to override any interest I might have in who's smarter or whatever.

I'm asian and I won't ever donate a penny to any of my alma maters because I was always treated as an uncool outsider.

Good on you, I don't blame you. Of course, objectively speaking, I'd say you were treated appropriately -- after all, you are an outsider and asians are pretty uncool (to non-asians; I have no idea how cool you find yourselves).

Silver

Fred said...

If universities want to increase NAM enrollment, why not switch to an admissions approach that rewards the virtues Cecil Rhodes prized:

1. literary and scholastic attainments;
2. energy to use one’s talents to the full, as exemplified by fondness for and success in sports;
3. truth, courage, devotion to duty, sympathy for and protection of the weak, kindliness, unselfishness and fellowship;
4. moral force of character and instincts to lead, and to take an interest in one’s fellow beings.


NAMs may lag on 1, 3, and 4, currently, but Asians probably lag on 3 and 4 too. So if NAMs step up 3 and 4, they could outshine Asians in aggregate, even if they perennially lag on 1.

Hacienda said...

"I think dutiful behaviour is a good thing, since it's the Conservative virtue, but it's important to consider this, or else you won't really understand the character of Asian civilization,"

Asian civilization lacks some critical ingredients, for example, Whites, Jews and blacks. If they existed in Asia, you would get more creativity than you want, much more. The most outrageous and vacuous Alpha/Beta/Omega male struggles. Apparently Asian females see through the illusion so want to keep things on the peaceful side, maybe white women do too, but there is enough of a "dumb broad" segment that Western culture admits of any kind of male competition lunacy? Or is pretty much helpless to control it. BTW, I think the Jew worship on this site is just preposterous. The Jews are victims in the West at least as much as victors.

dearieme said...

"The observed reality of White intellectual preeminence over centuries ..": well, about five centuries I suppose, unless you are referring to the brief flame of Ancient Greece. And for the last four of them, disproportionately British and German - and at the very highest level, it is noticeable that Americans are absent. Maybe they are just hard-working imitators of the Brits and Krauts.

beowulf said...

That's interesting. In other words, the College Board inadvertently began tested for conscientiousness.

Incidentally, the above comment is mine, I forgot to sign back in after a friend checked her gmail on this laptop.

JSM said...

"Yan, I'm pretty sure you love Chinese-Americans more than anything else in the world, and America comes in a distant second. But distinct second is still not that bad I guess..."

What Yannie loves is the idea of the United States -- the geography and the conveniently prebuilt infrastructure -- filled with Chinese (not Americans).

There's a reason he carefully chose the words "the United States" (the geographical region) as opposed to "America" (the land- and-its-people, the American-Americans).

JSM said...

"Don't bother sending your kids to Highschool.

Send them to Community College instead."

Hi, anon.

With just ONE caveat. Only do this with an extremely conscientious, self-reliant, good kid who is capable of directing his own learning.

Because there's this nasty little federal law with the Orwellian name of Family Education Rights and Privacy Act.

In it, if your offspring, of any age, is attending any "post-secondary educational institution," you as the parent will be DENIED access to school information regarding your child's class choices, grades, attendance records, or teacher opinions.

According to FERPA, if your minor child attends CC and you'd like to know how he's doing, you have the option of asking your kid. If your kid lies to you, oh, well.

On the bright side, though, they'll still be delighted to accept your check for tuition.

beowulf said...

Perhaps the only real answer is a real free-market in scholastic entry, where admission is openly and transparently won by test score and nothing else.
Perhaps the only real answer you like, you mean. Its as politically impossible as a real real free-market approach of just auctioning off admission slots to the highest bidder.

If universities want to increase NAM enrollment, why not switch to an admissions approach that rewards the virtues Cecil Rhodes prized:
They kind of do that now. As Leonard Sax (His book Boys Adrift is outstanding) has pointed out, its the reason elite colleges have an informal quota that benefits males applicants.
"If Princeton had gender-blind admissions, the Princeton student body would be 70% female," the director of admissions at Princeton recently told former Secretary of Education Bill Bennett. Most of the boys who apply to Princeton are good students with decent grades and test scores. But many of the girls are simply "amazing." Not only do the girls have straight-A's and great test scores, but they're also starting their own non-profit organizations, working at soup kitchens for the homeless, inventing new vaccines, etc.
http://www.rochester.edu/college/wise/resources/why.html

Hapalong Cassidy said...

Given the increase in the population of mixed-race Asians (like myself - 1/2 Anglo, 1/2 Korean), I wonder how these "elite" private universities plan on classifying them. I concede that most 1/2 Asians look Asian (although honestly I don't remember what I put on my college application), but most of them have Caucasian surnames. Also, every 1/2 Asian I have known (and there are more than you think) has married a Caucasian, myself included. So there is a growing population of 1/4th Asians out there. I will probably be putting "white" on my kids' college applications, as I'm guessing others will if they're smart.

Kylie said...

"I'm asian and I won't ever donate a penny to any of my alma maters because I was always treated as an uncool outsider."

It's "almae matres", not "alma maters".

Well, that was a pretty uncool outsider's mistake.

rob said...

This whole donating to your college is all about a way to justify discriminating against asians.

You must be asian. Whites were donating to their schools long before anyone gave a thought to how charitable Orientals aren't.

Zippy said...

What do you mean, dearleme? In terms of accomplishment in science, technology, and industry, the United States is at least the equal of Britain and Germany.

More broadly, if you looked at China and Japan, say 500 years ago and compared it to Europe, you could easily conclude that Asians were smarter and more on the ball. Then we had the Enlightenment, a whole scientific and industrial revolution, and the west pulled ahead.

Of course it's pretty clear in terms of civilizational competence that blacks, Australian Aborigines, and Native Americans just don't have what it takes. The real contest is between Europeans and Asians.

One theory would be that the amplitude of the distribution is wider among Europeans, so you get more dullards, but also more geniuses.

Another possibility is that the conformity of Asian culture -- which could have biological underpinnings -- means they can use their high intelligence to implement other people's ideas well, but they're less suited to coming up with new ideas.

A third idea is that it's just luck: the west pulled ahead for a variety of reasons, and that pulling ahead was just a temporary lead. Maybe the Chinese are going to kick our ass for real.

Of course, the one obvious response is the one that's politically untouchable: allow immigration of high IQ people from throughout the world, and stop low-IQ Third Worlders from coming here. South Africa is already exporting its high IQ whites -- I suspect quite a few of them would make great Americans. Britain itself is probably so far gone that their high IQ whites will want someplace to go. We should take advantage of those opportunities.

Georgia Resident said...

"So test-prep and studying is now bad?"
The point of an aptitude test (like the SAT) is to measure ability independently of preparation, so that one can see how cognitively well-equipped the student is, and thus how well they might do at college. Some tests, like Raven's Matrices, do this better than others. To a certain extent, however, one can prepare for any test, so they have to be periodically modified to preserve their g-loadedness.
Test prep is bad for society, even if it's good for individuals ("fallacy of composition" and all) because it both reduces the comparability of test scores (is he really that smart, or did mom and dad send him to cram school to raise his scores?) and leads to significant resources being spent on what is essentially a zero-sum game.
What would be best, in terms of guarding the g-loadedness and comparability of widely used aptitude tests, would be an absolute ban on any form of test prep, although that solution is not really feasible.

Anonymous said...

I'm a bit unclear on the relationship between the PSAT and the SAT. Does ETS write both? Has the NMSC eliminated analogies, too?

If the PSAT bears greater resemblance to the old SAT, then the sudden rise in the number of Chinese semifinalists seems likely to be the effect of increased elite immigration, rather than a growing population of ABCs. This would suggest the cause of rising Asian SAT scores, too.

Anonymous said...

"Maybe the reason Koreans are less likely to donate a few million dollars to have their name on a building is because they all have the same name?"


Maybe they are less likely to donate period.

Was there ever an organic democratic state among East Asians? Was there ever a tradition of volunteerism anywhere in East Asia? I am no expert, so I am asking.

Perhaps Asians, despite their intelligence and conscientiousness, still benefit from European organizational skills that draw heavily on cooperation, consensus and peer respect as much as Asians own respect for authority and hierarchy. The differences between NE Asians and NE Euros are slight yet significant and it seems that cooperation between them is more beneficial to both groups than interaction between any other combination of races.

Both Asians and Euros bring real assets to the table and the corruption levels are low enough to be manageable unlike many groups where corruption, violence and stupidity lead to severe dysfunction and almost entirely impede progress.

Anonymous said...

"it is noticeable that Americans are absent. Maybe they are just hard-working imitators of the Brits and Krauts."


Uh, Americans ARE largely Brits and Krauts, certainly among the thinkers, inventors and hard working imitators.

Anonymous said...

WN: "We don't need tests, history shows we're the smartest."

Roman proconsul in Britain or Germania: "I don't care what your tests say, Tacitus. These barbarians will never create a civilization to rival ours!"

Do you see what I did there?

Sheila said...

You worshipers at the altar of the high IQ and/or elite universities really are amusing. Just line up to get your ticket punches and you're assured of success and a happy life. My older kid's IQ is pretty much off the charts (yes, we had it privately tested)- that and a buck will get you a cup of coffee. "Mediocre" private schools? His high-rated public high school (filled to the brim with "smart" Asians and Jews) taught him nothing he had not already learned at his Christian elementary/middle school. Shakespeare? Check. All math up to calculus? Check. Basic physics? Check. Latin? Check. (Notice I don't include basic math and grammar which is a given, and which he had to "help" all his classmates with in college.)

As far as community colleges versus big name schools (I wouldn't know about "Kaplan online;" I wouldn't subscribe to any Jew's get rich on people's ignorance scheme), I'm unfamiliar with Caltech but can speak from experience regarding Harvard, the little Ivies, and the Seven Sisters. The hardest thing for a Harvard undergrad is surviving the competition for admission. After that, it's all TAs and grad students and month's long exam prep. The various grad programs differ in quality, but undergrad is a massive joke. If you're talking about basic survey courses and undergrad requirements (U.S. History, basic English composition, etc.), most community college courses here are taught by professors, some of whom are quite dedicated. The classes aren't as big and personal attention is more readily available. First two years of college at a fraction of the cost. Combine it with working in the real world and it's even cheaper - and your kid learns to support himself and exactly how big a bite the government takes to redistribute to blacks and hispanics (and all the Indian and Asian grandparents here on supplemental social security).

Intelligence matters, but so does drive and commitment and community, and diversity - even diversity with high IQ - destroys that. Thus I repeat:

Yan Shen, go home.

eh said...

Then there's the fact that Koreans, Chinese, and Indians are immigrating here in huge numbers,...

Yeah.

The rest of this is just SAT minutaie; I guess we'll all just concern ourselves with that while the country is overrun and demographically destroyed.

ATBOTL said...

"I'm asian and I won't ever donate a penny to any of my alma maters because I was always treated as an uncool outsider."

If you don't like being an outsider, you should have stayed in Asia.

Anonymous said...

Asian civilization lacks some critical ingredients, for example, Whites, Jews and blacks. If they existed in Asia, you would get more creativity than you want, much more.

The creativity of the West comes from whites, not the other groups:

mangans.blogspot.com/2010/05/how-did-we-ever-get-along-without.html

Anonymous said...

"I'm asian and I won't ever donate a penny to any of my alma maters because I was always treated as an uncool outsider."

Are you uncool?

Are you an outsider?

If so, why did you want to go to school there anyway when there are so many awesome Asian schools with arms open wide begging you come?

If you could be top of your class at a way better Asian school, why hang for four years with all those rubes who can't appreciate your awesomeness like your co-ethnics do?

Or maybe the bad old USA school is actually better AND more welcoming than anything your forbears built for you, if they built anything for you.

As for donating, don't make us laugh.

Nothing, and I mean nothing would make you donate. You aren't interested in supporting an institution for the sake of posterity not even your own. You expect us to do that. Shame on you.

What a joke.

Anonymous said...

dearieme: "well, about five centuries I suppose, unless you are referring to the brief flame of Ancient Greece. And for the last four of them, disproportionately British and German - and at the very highest level, it is noticeable that Americans are absent. Maybe they are just hard-working imitators of the Brits and Krauts."

Now, now, dearieme, don't be petty. Leaving the French out of the picture like that!The big four of Western Civilization for the last five centuries have been England, France, Italy and Germany.

RE: America,

Well, yes, compared to the Big Four America looks fairly inconsequential. Of course, the proper point of comparison would be other Western colonial outposts:Australia, Canada, Mexico, Brazil, New Zealand, Argentina, etc. Compared to its true cultural peers, America has an outstanding record of achievement.

Americans absent at the very highest level: That seems a bit excessive.Certainly, Thomas Hunt Morgan (biology), Edwin Hubble (astronomy), Orson Welles (cinema), etc. are commonly regarded as having done work that was at the very highest level in their respective fields.

Steve Sailer said...

"If the PSAT bears greater resemblance to the old SAT, then the sudden rise in the number of Chinese semifinalists seems likely to be the effect of increased elite immigration, rather than a growing population of ABCs."

I don't know the details either, but that's the kind of question we need asked to find answers to the question I'm most interested in right now: why the rise of Asian SAT scores over the last decade? In general, stasis is the norm on the SAT, but now we have a decade long trend that stands out from the stagnation that has been the historical norm for all groups on the SAT. That trend strikes me as pretty interesting, and we need people to identify naturally occurring experiments to get a better understanding of it.

Anonymous said...

Columbia University's engineering school is called the Fu Foundation School of Engineering and Applied Science after a Chinese businessman named Fu who made a donation to the school. When I first heard the name of the engineering school I thought "Fu" was "F.U." and were initials for something but it turns out it's the Chinese surname.

Kaz said...

@Anonymous 1:36

I'm not sure if that guy is actually asian or just poorly phrased his comment.

What he said rings true for a lot of white nerdy young men in college. Most young white men who are in STEM will be outsiders and uncool just as any asian male in the same field; unless you go to a school famous for it's STEM curriculum. It's all about football and partying in some schools, and you can bet your ass that nerdy young white men will be overlooked.

@Eh 1:00PM

If it helps you feel any better asians have a fertility rate barely above 2.0 in America, and that's ONLY due to the first generation asian immigrants. Also Korea has a ~1.2 fertility rate, Japan has a ~1.2 fertility rate, China will have a collapsing fertility/birth rate as well in the coming future thanks to the 1 child policy's ruinous effects on the male to female ratio (males significantly outnumber females in China).

@Beowulf

Haha..

@JSM

Jeez, come on man. A lot of people recognize America (North/South) is a damn continent and instead prefer to go with the US. It's probably something that comes from years of formal writing, you can't just say America in a formal essay.

Kaz said...

Also can someone tell me where this whole donating meme came from? Where's the data?

Do white people making 100k/yr donate to their universities more than asian people making 100k/yr?

Anonymous said...

Steve,

The discrimination against Asians in elite private colleges: Can you say.... (wait for it).... "Jews"?

Hacienda said...

Sheila:

"Christian elementary/middle school. Shakespeare? Check. All math up to calculus?"


Which school is this?

Hacienda said...

"The creativity of the West comes from whites, not the other groups:"

Whatever. I'll take that comment straight. My point was white creativity arose from a multicultural/multiracial/multireligious context. And if Asian culture was similarly blessed or cursed, I'm sure Asian Civ would be creative +++. So more annoyment from me at the "magic white" notions of creativity some posters here so lavish in, like a dream.

Kaz said...

@Shelia 12:53

Sorry I was not speaking of private schools for K-12; I meant private schools as in private colleges which is why I mentioned it while talking about BA's/BS's. Immediately after I made that post I wished I could edit that in to make it clear. Although I'm still not sold on the value of private schools if you're already living in a town with an excellent public school.
-
Now you're right about the basic courses for undergrad, going to community college for 1-2yrs to get your core done is a fine strategy if you make sure beforehand that the courses are transferable to your state university. But there is no reason to stop there, your state university probably charges the same tuition (although with some added fees) as a community college. So why not pursue a BA/BS? Now, I can't say this all with certainty since practices differ state to state.

Lastly, about Indian/Asian grandparents, on average Asians (that include Indians) have the highest incomes in America, thus they also pay a lot of taxes, also take in less services from the government, so don't you think they've paid their fair share at least? Regardless, why fuss over such a small portion of the country? Social security/medicare is killing us, I don't need to tell you the primary beneficiaries of those programs. Not to mention other government wasteful spending and corporate welfare. But it's okay as long as the person stealing your money is white and christian; or have you characterized them all Jewish by now?

Wunderlich said...

Hmmm. One wonders what Mitch's SAT score was. You know that very first one that more closely resembles ones actual IQ.

corvinus said...

Uh, Americans ARE largely Brits and Krauts, certainly among the thinkers, inventors and hard working imitators.

I.e., the white people.

Evil Sandmich said...

Foreign Asians, who don't qualify for financial assistance, are increasingly fashionable with colleges.

And high schools. Private high schools love them some foreign suckers who will pay the 'rack rate' to attend their institutions.

map said...

I think that you are missing a key ingredient: the privilege that immigration bestows upon the Asian of being able to arbitrage between two different cultures, essentially picking and choosing those elements that he actually wants and eliminating those that he does not.

Asians have the privilege of treating America as a dumping-ground for all of their money-making ambitions, while the social, cultural, religious, tribal and familial aspects of life are taken care of by the Asian portion of their lives. They do not need to work to become a part of American society because they already have a well-developed Asian society in which to partake and conform.

The same is not the case for Americans. Native-born Americans who do not have the option of emigrating are forced to actually make a life here. They cannot simply arbitrage between several different cultures and are, thus, forced to spend time learning and conforming with many cultural aspects beyond academics and test-taking.

What benefit is there for an American adopting some Chinese cultural practice to compete with people who don't belong here in the first place and should not be enjoying any privileges whatsoever? China does not grant me the right to exploit their peasants for the purpose of increasing my chances of snagging a hot wife and building a family. Why should America be some platform for Asian ambitions at the expense of the Americans trying to build a life here?

And when I say privilege, I mean it. The Asians are exploiting a gift granted to them by the US government that should be taken away.

Anonymous said...

My point was white creativity arose from a multicultural/multiracial/multireligious context.

You might want to check the link again:

mangans.blogspot.com/2010/05/how-did-we-ever-get-along-without.html

There's nothing "multicultural/multiracial/multireligious" about it. It's white Christian.

Kaz said...

@map

Are you kidding? China's peasants are 'exploited' by corporations around the world.

I don't even get the rest of your post, it's all assumption based on your falsities.

map said...

Here's an example of how this cultural arbitrage operates.

In America, a white man born, raised and educated here is considered a loser if he lives at home with his parents beyond a certain age...like 22 years. It is a common cultural practice for white American men to be expected to be on their own paying their own way...or, at least appearing to pay their own way.

Now, in India, it is a common cultural practice for adult children to live with their parents up until they are married...or even after they are married.

So what happens if an Indian man brings his family to America and retains the cultural practice of living at home with his parents or other members of his family? He gains a huge advantage. He avoids a cost that native-born white men have to bear as a condition of making a life in America because no, say, Indian woman is going to complain about an Indian man living at home with his parents. Such households can have 3 or 4 different incomes supporting them.

This is the kind of cultural arbitrage that immigration brings and that disadvantages white Americans. Mexican laborers can live 20 people to a single-family house. Americans don't want that. Asians can have their kids studying 12 hours a day because all the other Asian kids are doing the same thing. American parents don't want that because there is no peer group doing the same thing.

Americans individually adopting foreign practices results in becoming a weirdo.

Mitch said...

I agree with Map that Asian immigrants here are picking and choosing cultural elements. I teach at an SAT prep academy for Asians (among other jobs) and all the kids' parents have every intention of going back to their country of origin. They are here to give their kids cheap, excellent educations with far less competition, and then they're going to go back home. They have no intention of adopting any aspect of this culture, and don't expect their kids to, either--except where it's advantageous. I'm speaking generally, but it's well over 50% of recent East and South Asian immigrants to the California area. I think I've posted this before--every Indian student I've worked with has readily told me that their parents expect to arrange their marriage, and that they are "kind of hoping to talk them out of it". I'm relieved they aren't happy about it, anyway.


On the PSAT--it's identical to the new SAT; just fewer questions.

ZZ said...

--Ron Unz's research showed that 20 percent of HYPS is Asian, and 25 percent to 33 percent is Jewish--

That estimate for Jews at HYPS is way too high. Maybe at their peak in 1975-1985 that might have been close, but the numbers are much lower now due to intermarriage, delayed childbearing, and low birthrate.

Now maybe if you want to include ethnic half and quarter Jews who don't identify as Jewish their number at HYPS might be around 20%, but just going by names and looks 10% of the class is more like it.

If you are curious, Harvard will give you the names of every student by year in its dorm history website. Freshman dorms are basically assigned randomly. So just typing in a year and looking at freshman buildings, you see a random list of freshman names.

Here is the most recent list available:

http://www.hcs.harvard.edu/~dorms/index.cgi?name=&grad=2011&year=&dorm=-+Any+-&room=

As you can see, well under 10% of the names there appear to be Jewish, and the large majority are white christian last names with non-black first names.

Anonymous said...

"well, about five centuries I suppose, unless you are referring to the brief flame of Ancient Greece."

I wouldn't call the period between Eudoxus and Diophantus brief but I guess to each his own.

Anonymous said...

"Zippy said...More broadly, if you looked at China and Japan, say 500 years ago and compared it to Europe, you could easily conclude that Asians were smarter and more on the ball"

Is this a joke or do you really have no idea what Europe vs Japan looked like 500 years ago?

Anonymous said...

"In fact, if we changed the question from "does the SAT/ACT predict college success better than the subject tests or GPA" to "does the SAT/ACT predict college *readiness* better than the subject tests or GPA", we could all go home, because that question has already been answered. "

The SAT predicts ones readiness to major in math or philosophy, but as we all know, elite universities provide an abundance of easier paths for less academically gifted NAMs and legacy kids to successfully tunnel through four years of college. Universities also engineered grade inflation to keep students from wondering whether its worth paying $35,000 a year for 2 C+'s and a 2 B-'s.

The true value of the SAT/ACT to universities is its predictive power of alumnis post graduate earnings and professional success. And when they interpret SAT scores they are also factoring in the real and positive impact of melatonin due to rampant affirmative action. It's probably worth the effective 200 point handicap which colleges give to blacks and Hispanics.

Kaz said...

@Mitch

What? 50% of them want to go back to their country of origin..? You're happy that Indian men are forgoing arranged marriages? Why? So we're all forced to marry sluts?

You know what, I'm sick of posting here.

Cecil Rhodes said...

Fred said...

If universities want to increase NAM enrollment, why not switch to an admissions approach that rewards the virtues Cecil Rhodes prized:

1. literary and scholastic attainments;
2. energy to use one’s talents to the full, as exemplified by fondness for and success in sports;
3. truth, courage, devotion to duty, sympathy for and protection of the weak, kindliness, unselfishness and fellowship;
4. moral force of character and instincts to lead, and to take an interest in one’s fellow beings.

NAMs may lag on 1, 3, and 4, currently, but Asians probably lag on 3 and 4 too. So if NAMs step up 3 and 4, they could outshine Asians in aggregate, even if they perennially lag on 1.


Any contemporary or historical comparison between NE Asian countries and NAM countries of origin in Africa and Latin America expose Fred's delusional anti-Asian reasoning.

For anyone aware of history beyond the past few centuries and aware of current trends, the obvious comparison and "coopetition" lies between the West (Middle East->Europe) and NE Asia centered around China.

Number 2 above is the only point that isn't patently absurd, but largely because it's vague and even senseless. For example, having the "energy to utilize one's talents to the full" has little to do with chasing balls in spandex in the real world.

While sports can teach teamwork, leadership, toughness, etc they are completely artificial worlds of entertainment that act as very distant proxies for real world situations like military or economic competition.

There are far more realistic venues for individuals to learn and exhibit these virtues such as high tech startups, complex project management or large-scale organized modern warfare which is increasingly reliant on technology and automation.

Certainly civilizations gain relatively nothing from the transient cultural contributions of sport entertainments like who was the 1978 Cy Young Award Winner.

Steve Sailer said...

Gaylord Perry

Jack said...

If not for athletics, there would be few white gentiles at top universities. Some schools have 30 teams. Not a lot of Asian recruitable athletes, so vast majority of the athletes are white. These athletes can get in with at times much lower academic stats. But it's a way to keep the schools at least majority white.

If Asains are actively discriminated against on their own, controlling for athletic and legacy status, it's not by much. And to be honest, they probably should be discriminated against. White Americans built this country and these universities. Asians expect to come here, score high on a test, and automatically walk into the elite. Doesn't work that way.

Anonymous said...

This place is becoming a lot like stormfront.

Ok, so asians do better in school, do much better in mathematics, become scientists, inventors, and doctors at a higher rate than white people.

But they are bad for society, because I don't like the look of them, and they are too polite(boring) for me.

Anonymous said...

I think the estimates of Harvard being 25-35 percent Jewish are probably counting 1/2 and 1/4 Jews. In the elite these days, probably a significant fraction do have some Jewish ancestry.

I'd also bet that a lot of elite whites, outside of the east coast, are okay with their children attending a regular State U over an Ivy. In the more prestige-obsessesed east cost cities, in which the majority of elite whites are probably of some Jewish ancestry, there's a much stronger desire to see a child go to Harvard. A white gentile businessman from the Dallas exurbs might be fine with his son going to Texas A&M, while a wealthy New York father (of any race) wants his son to aim for an Ivy. This tends to result in fewer gentiles applying to these places.

JSM said...

Hey, Mitch,

I'm out here in flyover country with one of those brilliant boys that the random genetic recombination of us Germanic farmer-types occasionally produces.
(Not too many Asians around here. Oil drilling is too strenuous.)

My non-tiger-mother method of prepping him has been to co-enroll him in community college.

He's about to take the PSAT. Any advice?

Not that I WANT him to go to HYPS. I just want him to be *admitted* so we can use that fact to negotiate a better financial aid deal with western State U's.

Thanks from the housecat-mom of a kid bearing the brunt of Asian immigration.

JSM

Anonymous said...

He's about to take the PSAT. Any advice?


1.) Get as high of a score as possible. If he scores in the 99th percentile, he could be eligible to be a National Merit Scholar. That would be really good for him.

2.) Study up big on the math part. The math is pretty easy and, through sufficient drilling, he can ace it. Reading is a little tougher, but try memorizing vocabulary words.

3.) Don't obssess over one question on the test. Missing a few is no big deal, if you can get the large majority of them.

4.) Take a few practice tests first. It'll make him more calm and more prepared to do well. He'll also know what to expect and move more quickly.

Disappointed Asian Reader said...

It's amazing to witness the extent white people are going to rationalize their superiority over all the other races.

I'm quite disappointed to read the responses of commenters once Sailer began covering Asian-white testing gap. It's the opposite of what I thought was the spirit of the HBD-sphere (intelligence being a desirable characteristic, transcending race. If a black man has an IQ of 140, he is as good as any other man with an IQ of 140, etc).

But recent events at Sailer's blog has only confirmed my hesitations about the dangers of the information of HBD. It's going to evoke people's inner racism (the bad kind of racism, not the realist kind).

What did Asian-Americans ever do to white America to generate hostility? We work hard and do what we are supposed to do yet we are never going to be good enough. You guys are digging deep to strip Asian-Americans of our recognition.

I think it's a big mistake to paint this as a competition between races. You guys fail to see the potential partnership between us. This is especially important given how NAMs and liberals are working in tandem to take power from both groups.

As far as Asian underachievement goes, here is what I have to say:

- There may not have been great discoveries or inventions but Asian civilization flourished and was stable for thousands of years. What is the goal of civilization if not to create a secure society with bounty for most? Discounting the modern era, can Europeans say the same?

- You guys are quick to forget Asian-Americans are a fairly new cohort in American society. Most of us are 1st or 1.5 generations in. Not counting legacies, most people do not enter upper levels in their industry until they are in their late 40s or 50s. Give us time to age.

More importantly, the Americanized generation of Asians (such as myself) will adopt the best traits of Western culture and integrate with it their mother culture. This will put us in a better position than our ancestors (high IQ + optimized culture).

I suggest white Americans who feel threatened should be ready to adapt traits that can give them the best advantage as well. Otherwise, you know, natural selection will simply run its course.

Mitch said...

JSM,

I disagree with anon@2:34's advice.

1) As I mentioned earlier, the SAT is shorter since 2005. That goes, if not double, than 1.5 times for the PSAT. The impact of each missed question is considerable, and any major player for the top scores must get every question right, or as close to it as possible. If your son is currently struggling more than 1-2 questions per test (struggling with content, not making unforced errors), then he's not in top territory (which is nothing bad, I'm just telling you what the state of play is).

2) Understand that the writing test (no essay on the PSAT) *is* out to get you. Understanding how the questions are constructed and what to look for is essential.

3) For both math and writing, a good way for top students to get better is to drill, but *not* entire practice tests, which is a complete waste of time. Get the blue Official College Board book, or one of the Kaplan big books (not Barrons. Never Barrons) and drill sections at half time.

So for a 25 minute math section, give him 15 minutes and tell him to work as fast as he possibly can (without rushing), skipping any question that he doesn't instantly see how to do. in 15 minutes, a top tier student should be able to finish an entire math section with at most 1 question he's not instantly sure of. That leaves him 10 minutes to spend more time on the 1 question he needs to get done, and then check his work. Have him practice checking his work--I used to do sections for my kids with errors in them and have them spot them.

Ditto for writing, except there won't be any difficult ones, and he has to make sure he's seen certain types of questions (or he's missed it). The most common missed question on the writing section is a "no error" on the ISEs.

4) Reading: best way to improve reading is to stop reading the passage entirely the first time through, but read for organization of ideas. Do transition scans, read first sentence of paragraphs, get a sense of the big picture. In my experience, most top kids struggle with at most 4-5 questions on the whole test, and reading for organization will help them see how the passage is structured.

He should be spending up to 50% of test time sitting around, having completed entirely and checked his work for unforced errors.

One caveat: every so often a top kid is highly structured and this method will throw him off his game. But if he's highly structured, he's not making that many mistakes anyway. If he is, then it's content, and it's a different problem--but again, he's not topranked if he's missing a lot due to content.

I'm assuming you know that, other than the National Merit aspect, the PSAT has zero value.

Don't forget the ACT; many top students have a strong preference for one test or the other. My son picked up 150 points (converted) because he was so much better at the ACT.

Anonymous said...

"I agree with Map that Asian immigrants here are picking and choosing cultural elements. I teach at an SAT prep academy for Asians (among other jobs) and all the kids' parents have every intention of going back to their country of origin. They are here to give their kids cheap, excellent educations with far less competition, and then they're going to go back home. "

Yes, but their kids are likely to stay so the parents will, too. The longer an Asian lives in the US, the harder it is for them to reintegrate into the rigors and conformity of Asian societies. Remaining in the East Asian ghettos of the South Bay or the San Gabriel Valley is far more congenial. Feelings of home sickness can be instantly dispelled by a trip to the local Ranch 99, preferably entering through the back entrance next to the dumpsters, which provide all the olfactory delights of home. The occasional money order wired to an aged relative or less fortunate cousin left behind is usually more than enough to allay any feelings of guilt. And of course there are the annual pilgramages back to the old country to insure that the US is kept stocked with freshly mutated flu viruses and pesticide resistant bed bugs.

Anonymous said...

I fail to understand Steve Sailer's basic premise here, that somehow if it were the case that cramming was why Asian scores were increasing, that this would constitute "unfairly gaming" the exam. Since when did studying harder than someone for something violate accepted principles of fairness? This is an absolutely terrible value to impart upon your kids. "Don't work too hard at something, otherwise you might be unfairly gaming the system!"

The bottom line... Steve Sailer makes zero sense here either way, regardless of the reason for rising Asian American SAT scores. If the typical Steve Sailer thesis is correct, that Asian conscientiousness explains their high SAT scores more so than innate intelligence, then whites only have themselves to blame for being lazy. The more interesting question to ask is, assuming that the Steve Sailer thesis is correct, does conscientiousness have a genetic basis? If so, what difference does it make then whether or not an achievement gap is due to a difference in innate intelligence or innate conscientiousness?

Anonymous said...

SUNSHINE is an interesting movie in this regard, not least because its cast is made up of 3 East Asians and 4 whites(and 1 Maori dude).

All the people are very competitive but not for individual reward but in being right on the matter of what's good for the mission.
Each person can be ruthless but in being selfless. The thing is they're as ruthless in self-sacrifice as in sacrificing others. And they overlook personal vendetta in choosing who should live and die too.
This explains why some people--Japanese, Germans, etc--have achieved more. By nature, people wanna win for self-gain, and of course, this is necessary for capitalism. But for any enterprise to succeed, its members must be willing to accept what is good for the entire group and mission, even if it means less for themselves. So, Germans want the 'better German' to win the top position and will loyally serve the 'better German'. The 'better German', appreciating the good will of other Germans(who play by rules), will be more willing to share his rewards with them. In Greece, on the other hand, a 'worse Greek' will try to rise over 'better Greek' through all sorts of dirty tricks. Even in America, a Greek mother will use her muscle with the teacher to get her idiot son into a honors program, the bitch.
Same in Sicily according to Puzo's 'The Sicilian'. If some moron who can barely read a book wanna a doctor, he goes to a mafia boss who pressures the medical school to make him a doctor. And Negroes... oh well. The fact that someone like Obama will so shamelessly feel entitled to being president though he hasn't done shit tells us something. For all his talk of post-racial America, he is a typically self-centered black guy.

There are certain cultural types in SUNSHINE.

The Japanese guy accepts his death like a stoic death-cult samurai: in a way, he seems to even welcome it.
The chinese woman(played by michelle yeoh)is like Amy Chua. Her mind is on oxygen, oxygen, and oxygen. And since there isn't enough, she says 3 people must die(or be killed).
It's a kind of inhumanity, but since each is willing to give his or her life for the common good, it is a kind of humane inhumanity.
Another Asian guy(trey)is very much the 'me so sorry, it all my fault' kind of Asian.

Among the white guys, there is Mace who is very efficient, action-oriented, and even gungho. He knows what must be done and will do it.
There is Capa, the David Bowman of the group. He even looks Starchildish. He's like the white guy with larger view of things.
There is Cassie, who turns out to be something of a softie though she knows what must be done. She's SWPL-ish in emotions. She really can't think ruthlessly after all.
There is Harvey, who looks Southern-European(or maybe Jewish) like, and turns out to be less self-sacrificing than he's supposed to be.

And then, there is the Maori-actor Searle who's ruthselfless too.

Though the premise is scientifically ludicrous(it seems lifted from the last episode of the anime GUNBUSTER), the movie makes pretty impressive use of technology. It's not just about speed and explosions but how things look and work. And I like the designs.
Not as good as TRON LEGACY or conceptually interesting as INCEPTION, but still one of the best sci-fi in the last 20 yrs.

Anonymous said...

"Actually the huge mountain of evidence favors superior white intelligence: It's the historical record. Whites have produced wildly disproportionate intellectual contributions vs. Asians in almost every field of human endeavor over the past, what, 2000 years. The observed reality of White intellectual preeminence over centuries is a lot more than "noisy hand-waving." "

Perhaps in comparing whites and Asians we are getting means mixed up with dispersion. The whites produce more over the top creative people, while the Chinese on average produce somewhat better factory and office workers. Higher white levels of empathy and sociability mean they create friendlier and more pleasant communities capable of cooperation and philanthropy, albeit less orderly ones; the sight on Youtube of 20 whites lifting a burning car to save a trapped motorcylist in Utah comes to mind.

Perhaps whites and Asians should be kept apart since there can only be friction and resentment between the two groups when they are blended. Same goes for ALL the races. By now every ethnicity living in America has grasped that diversity has jumped the shark, and that nonwhites are here only to make a blatant grab for the white man's living space and resources.

Zippy said...

This comment thread really does demonstrate some of the less attractive aspects of the Stevosphere -- attributes not found in Steve's own thinking, I might add.

First, this need to denigrate America. The left is constantly harping about how America sucks -- Michelle Obama's comment about being proud of America only after her hubby got the nomination for President comes to mind. Lots of you guys seems to be saying "yeah, we really do suck, just not for these reasons." The idea that the US contributed nothing while Germany, France, England and Italy (!) were the real drivers is just ludicrous.

Look at the list of Nobel Laureates in real sciences. The US is well-represented. Consider inventions like the light bulb or the semiconductor. United States. Innovation, industry, we've done it all. Now it does seem we're not doing it now, but there is nothing wrong with America that isn't fixable.

Second, this anti-Asian bias. I'm sorry, but Asian-Americans really do contribute to this country, and for the most part they do assimilate. Granted, there are some attempts to turn them into whiney victim groups, but given their ability to succeed on the merits (and the price they pay in racial preferences), they ought to be natural allies in the fight for color-blind policies.

Finally, the economic zero-sumism. The left has this problem, too, but jeesh. The economists aren't right about everything, but they are right about some things. Gains from trade are real. Productive people do produce and that makes us all better off.

Anonymous said...

Mitch lost a lot of credibility with me here. He may be an OK cram school tutor but he's not such a great cultural observer. I think he sees what he wants to see (as most commenters do on this blog), and that calls into question his observations about SAT scores as well.

"... all the kids' parents have every intention of going back to their country of origin. They are here to give their kids cheap, excellent educations with far less competition, and then they're going to go back home."

In my experience the parents of the vast majority of Asian-Americans kids are unlikely to return to their own country. (Perhaps the sub-population Mitch sees at the cram school are not representative. But then his remarks don't shed much light on aggregate Asian scores.) In any case, the parents are old. It doesn't matter what they do. They kids will stay because they have been Americanized.

Mitch, have you ever collected any statistics on student performance at your school? Because no academic study has ever replicated the large gains claimed by test prep services.

Anonymous said...

Mitch lost a lot of credibility with me when he tried to claim that making the SAT math test harder would benefit whites. While I can see a harder reading section benefiting whites, it's hard for me to see how making the SAT math section harder, along the lines of the AMC or AIME, would actually close the Asian-white gap, as opposed to increase it.

I'm surprised that Steve pays so much attention to what Mitch says. I assume this is probably because Mitch just tells Steve what he wants to hear.

@Anon

There are plenty of studies claiming that on average extensive preparation for the SAT does little to boost scores, that the SAT is fairly g-loaded. Or even better, to quote Steve Sailer himself, if you outsmart an IQ test, that usually means you're smart.

Hacienda said...

"If so, what difference does it make then whether or not an achievement gap is due to a difference in innate intelligence or innate conscientiousness?"

Big difference. If it's innate intelligence, then end of ballgame. The entire "white world" needs a fundamental restructuring if it's to survive as "white" world. Something which seems to concern Steve, AFAICT. Maybe not.

If it's innate conscientiousness, then there is all kinds of fiddle room. Like whites have more fun, are more sociable, more inventive, the SAT is bogus, etc.

In the first situation the onus is squarely on whites. In the second the onus is on yellows.

Anonymous said...

Beowulf

"If Princeton had gender-blind admissions, the Princeton student body would be 70% female,"
"But many of the girls are simply "amazing." Not only do the girls have straight-A's and great test scores,"

I can't find the source for it, google search gets me to this:

http://www.singlesexschools.org/adboys.html

and a ppt titled "His is bigger-but hers is better connected"

with the source link dead. Maybe they cover it up really well?

The line isn't in Sax's book, neither in your link.
I have come across some articles that pointed out the affirmative action for boys, thought w/o many numbers iirc.

otoh there's one here:

http://www.yaledailynews.com/news/2008/nov/11/gender-gap-widens/

"When last year’s admissions cycle was over, the difference was clear-cut: 9.8 percent of men — and just 7.5 percent of women — were accepted. "

And the first comment:

"For the same reason the male admit rate at MIT is 8%, and the female admit rate is 20%. "

Searching for MIT's rate led to:

http://tech.mit.edu/V127/N14/admissions.html

"Those numbers show that the acceptance rate for women was nearly 26 percent for women and 10 percent for men. The acceptance rate for all applicants was 13 percent, with the number of male applicants nearly tripling the number of female applicants."

Now see, this is where the hilarious part starts.

"Jones said that the higher acceptance rate for women reflects fewer female applicants."

Which btw is the same argument used to explain some of the male advantage on SAT.
Less boys take it --> Less mediocre boys to compute boys' average --> Higher boys score

QED.

Now what happens when we change the roles?

OMG!! Discrimination, Patriarchy, Sexism, Self-Esteem, Romance, Larry Summers, Old Boys' Network yada yada, yada.

(contd.)

Anonymous said...

And to come to the punchline, (Miss?) Jones of the sagely wisdom above was found guilty of some credential fudging in 2007 and promptly(?) fired. (and I am presuming that you definitely know about Larry Summers fiasco, and the Nancy Hopkins breakdown.)

http://www.usatoday.com/news/education/2007-04-26-mit-admissions-dean-out_N.htm

http://www.nytimes.com/2007/04/27/us/27mit.html

And a peek during the good times before the fall:

"The issue hasn't always been at the top of Jones' agenda. A scientist by training, she was hired by MIT's admissions office in 1979 to help recruit more women, then just 17 percent of the student body. By the time she was appointed dean in 1997, that figure had grown to 39 percent.
Now, nearly half of MIT's incoming undergraduates are women, as is its president."

(with funny consequences:
http://isteve.blogspot.com/2011/03/intended-consequences.html)

Bravo, bravo. See, credentials ain't everything!
Though to her credit, she was not afraid to admit:

"But as that issue's urgency faded, Jones began noticing some other troubling trends in admissions.
... You don't see the kind of wild innovation from individuals you used to see," Jones said over lunch during a recent interview. "You see a lot of group and team projects overseen by professionals, but you don't see the kind of rogue, interesting stuff that we used to see at MIT.""


http://mitadmissions.org/blogs/entry/marilee_jones_in_the_news_1


and whereas your comment had laments are about girls with "amazing" resumes are being unfairly left out, she saw perfectionism as a problem.

"The quest for perfection "is making our children sick," the MIT dean of admissions told a recent gathering of college admissions professionals in Boston. "

(Or to do an Hegelian anti-thesis, she's applying Sailer's Law of Journalism in a very twisted way)

Maybe we should wait when video gaming prowess starts becoming a worthwhile qualification point in more fields than just hacking and computer science, so that these skills are also seen as worthy like that of helping the poor, and having good grades.
Hopefully other good women like Miss Jones will help change the perception.

http://boss.blogs.nytimes.com/2011/06/24/why-women-have-an-advantage-in-technology/

Mitch said...

You know, there's so many different threads of conversation going on in this thread that those of you who are characterizing it as anti-Asian, anti-American or so on really should stop tarring everyone with the same brush.

Our educational system is reeling from the sucker punches of a multitude of unintended consequences.

First, the admissions system began talking down standardized tests. They're bad, they reward prep and gaming, blah blah blah. Why? Because, of course, blacks and Hispanics get lower test scores.

So they started talking up the importance of grades, because in schools with majority URM students, grades, particularly in AP classes, could be relative, and schools could give them good grades. Colleges could then weight grades more than test scores (the UC system, last I checked, weighted grades a shockingly high 75%), and this would give URMs an advantage without really using "affirmative action".

This decision was, by itself, very harmful to smart kids in suburban schools who don't prioritize grades--not knowledge, but grades--above everything. It was particularly harmful because the importance of GPA led to increased use of AP classes, and the majority of schools gate AP classes with grades. So a smart kid who didn't get good grades was hurt twice--first, by not having As, second, by not having weighted As because they were denied access to AP classes.

Obviously, this hurts non-Asians boys a lot, white boys most of all, but it hurts bright kids who just don't care about homework in all categories.

And of course, the people who tacitly rigged the system forgot about Asians. They were mostly thinking about whites, because whites have a disproportionate number of high scorers with less than stellar GPAs. But Asians of all ability levels are intently focused on grades.

When people look at the overrepresentation of Asians in colleges, they think it's test scores. It's not. For the most part, it's the emphasis on grades, instituted to help URMs, that led to the floodgates.

As Steve said, it's quite possible that Asians are undesirable to colleges because of the donations. But I believe that the reason colleges feel justified in discriminating against Asians is that colleges didn't *want* Asians in higher numbers. They were looking for URMs, and the onslaught of Asians is something they can't stop without eliminating their ability to get URMs as well.

I think Asians know this, even subconsciously, and they realize that they are competing primarily against other Asians, which is why they are so ferocious about every point, even though Asians, more than most, would be aware of the weight given to grades.

Asians are doing nothing wrong. I don't disapprove of their actions, nor should my comments be interpreted as such. But the tests were designed for a population of America teens, not Asian teens living under far more demanding cultural rules, and on the edges, that will make a difference in test scores in the margin. However, the fact that Asians are doing so well in tests is only a small part of the reason Asians are overrepresented in universities, which is in large part due to their grades. Remember, it's not just that good grades give you a better GPA, it gives more opportunities that are now shut out to otherwise smart (and often just as smart, or even smarter) kids who just don't care about grades.

Our system is not designed for the onslaught of determined, smart, Asian immigrant parents who want a cheap education for their kids without actually becoming American. That does *not* mean they are doing anything wrong. It's our system that needs to be redesigned--not just our educational system, but our immigration system as well.

Mitch said...

Anonymous 8:20,

I'm not a guy.

Second, Anonymous 7:26's very funny post about what people do and what they think they will do is very much on point. (I live right near a Ranch 99, by the way, and they have great prices and excellent fast food. Just avoid the dumpster.) I was not talking about what the parents *will* do, merely what they *intend* to do, which is relevant when you think about Map's point. These are not people who have any intention of becoming American. Nor did I say their kids would go home. The issue, as I understood Map's point, is whether or not the immigrant population is interested in assimilating, and their intentions are relevant.

As for the data on test prep scores, all research shows that while the *average* gain is small, the range of gains is extraordinary. And I've worked for two test prep companies, but I am referring to the research, not their data.

JSM said...

Mitch,

Thank you so very much. If my kid does get NMS, he can get a full ride to ASU, I think. Your advice may be worth 20,000 bucks or more.

I can't express my gratitude for your generosity.


JSM

Anonymous said...

How did I left this out?

"Jones hopes someday to see MIT make standardised tests like the SAT optional for applicants. A growing number of colleges have stopped requiring standardised tests, though none of MIT's reputation, and for MIT to do so would send shock waves through the field."

Anonymous said...

Steve Sailer still hasn't explained why he thinks Asians studying really hard for a college entrance exam violates accepted principles of fairness(test prep as he calls it). Normally, the only kinds of people who make these arguments are blacks and Hispanics.

Mitch said...

"The SAT and ACT are, relatively speaking, very good tests, but we live in a highly competitive, globalized world full of people who are smart, hard-working, and less trusting and less believing in Fair Play than naive Americans."

Do you live in an area that is greater than 50% Asian, with schools that are 80% Asian? If so, then we can have a meaningful discussion.

If not, then you don't know what you're talking about and there's no point.

Anonymous said...

Beowulf(where did this comment go?)

"If Princeton had gender-blind admissions, the Princeton student body would be 70% female,"
"But many of the girls are simply "amazing." Not only do the girls have straight-A's and great test scores,"

I can't find the source for it, google search gets me to this:

http://www.singlesexschools.org/adboys.html

and a ppt titled "His is bigger-but hers is better connected"

with the source link dead. Maybe they cover it up really well?

The line isn't in Sax's book, neither in your link.
I have come across some articles that pointed out the affirmative action for boys, thought w/o many numbers iirc.

otoh there's one here:

http://www.yaledailynews.com/news/2008/nov/11/gender-gap-widens/

"When last year’s admissions cycle was over, the difference was clear-cut: 9.8 percent of men — and just 7.5 percent of women — were accepted. "

And the first comment:

"For the same reason the male admit rate at MIT is 8%, and the female admit rate is 20%. "


Searching for MIT's rate led to:

http://tech.mit.edu/V127/N14/admissions.html

Those numbers show that the acceptance rate for women was nearly 26 percent for women and 10 percent for men. The acceptance rate for all applicants was 13 percent, with the number of male applicants nearly tripling the number of female applicants. Data from the classes of 2008 and 2009 showed similar trends, and can be found in the table below.


Now see, this is where the hilarious part starts.

"Jones said that the higher acceptance rate for women reflects fewer female applicants."

Which btw is the same argument used to explain some of the male advantage on SAT.
Less boys take it --> Less mediocre boys to compute boys' average --> Higher boys score

QED.

Now what happens when we change the roles?

OMG!! Discrimination, Patriarchy, Sexism, Self-Esteem, Romance, Larry Summers, Old Boys' Network yada yada, yada.

(contd. 9/18/11 8:56 AM)

Anonymous said...

And Mitch, shame on you for complaining about Koreans in Korea gaming the SAT, which isn't even in their native language! I'd like to see white Americans take a Chinese college entrance exam in large numbers.

If foreigners taking the SAT in a non-native language can outgun native English speaking Americans(two out of the three sections on the SAT, the reading and writing CLEARLY favor native speakers over non-native speakers), then I have to say that white Americans should truly be ashamed of themselves.

Anonymous said...

Mitch,

Sorry I got your gender wrong.

"When people look at the overrepresentation of Asians in colleges, they think it's test scores. It's not."

This is a UC-centric remark. Private schools like Stanford, MIT, Caltech, etc. are not using the same admissions procedures and I suspect they are smart enough to know that high school grades load more on conscientiousness than IQ. Yet, those schools have large Asian populations.

Re: statistics, if your answer is that most Asian kids at cram schools don't raise their scores by very much, but occasionally some kid does (this is exactly what the academic studies show, including hugely well-powered ones done by ETS), then again I wonder whether you are really answering Sailer's question about aggregate scores. It's a nice story (esp. warms the hearts of WNs) that this yellow wave of workaholics are inflating their scores, but in fact are totally mediocre. (Hard to believe if you know anything about math olympiads and more g loaded stuff.) But is it true?

Slightly off topic: Many of the kids who raise their scores a lot may actually be getting an *educational* benefit from the cram school. If you're a free market type, you can easily imagine that the actual instruction (about subject matter, not just test taking tips and tricks) could be better in a private cram school than in a public school. That is, kids may actually *learn* things (like what the sine of an angle is) through the cram school. If that's the case, why complain?

Anonymous said...

"Do you live in an area that is greater than 50% Asian, with schools that are 80% Asian? If so, then we can have a meaningful discussion."

What fraction of Asian American kids are attending such schools, as opposed to those attending schools where they are a small minority? (Monterrey Park and Cupertino aren't exactly representative, even for Asians.)

If you can't answer this you don't know to what extent your observations have anything to do with Steve's question about aggregate Asian performance on the SAT. It might be "respected commenter Mitch" who is living in a bubble.

Mitch said...

Various Anonymi:

I'm not complaining. I have, on several comments in this thread, made observations that directly contradict the opinions you impute to me. I'm really uninterested in whether your determination to say otherwise makes you ignorant or dishonest.


PS--the SAT doesn't test trig.

Kylie said...

"Steve Sailer still hasn't explained why he thinks Asians studying really hard for a college entrance exam violates accepted principles of fairness(test prep as he calls it)."

Where did Steve write and/or say that? Link, please.

Zylonet said...

@Disappointed Asian Reader said...
"It's amazing to witness the extent white people are going to rationalize their superiority over all the other races."

Yes, it is ridiculous. Asians are smarter and more communal. Only idiots doubt that. However, you are somewhat confused about how people actual work, so let me explain a bit. Maybe you can then have empathy for people.

"What did Asian-Americans ever do to white America to generate hostility? We work hard and do what we are supposed to do yet we are never going to be good enough. You guys are digging deep to strip Asian-Americans of our recognition."

You make the same mistake as those who proclaim "white superiority." There is no "Team Asia" and there is no "Team White." An Asian doesn't deserve recognition because he is of the same race as people who have realized success. Same for white people: they are not part of a team and they deserve no recognition as a member of a team who has been successful. We are all individuals and we are basically alone in the world.

You ask what have Asians done to whites. Of course, Asians have done nothing wrong, but because of the workings of the mind, and most nearly everyone's desire to be around people of their own race, the presence of Asians results in a diminution of the value of the individual. By that metric, the mere presence of a mass of Asians results in a profound change to society. Because of divergent outcomes in all manner of life (academics, leadership, sports, sex), the individual is now thrust to the background while racialism is thrust to the fore. So if you are a white person who is sick and tired of all the problems of racialism and just want a nice, easy life, then Asians, by their mere presence and normal activities, are a cause of disruption.


"I think it's a big mistake to paint this as a competition between races. You guys fail to see the potential partnership between us."

Who is "us?" Do you wear a uniform representing Team Asia? You are painting the world like a liberal Godfather, as if the movement of your hands can dictate events between races. Whites do not act as if they are part of a team that is why there is AA and mass immigration. For that reason alone there can be no partnership. Most whites want some measure of individuality, and the notion of being part of "Team White" is unacceptable and disgusting. Count me among those who vomit at the notion of being part of "Team White." I am a white male; I am for ending mass immigration; I know that multiculturalism is lie. However, my best friend is Afghan and my wife is half Asian and my favorite co-workers are Brazilian.

continued below

Sheila said...

Hacienda - I'm unwilling to name the specific school (for reasons of personal privacy) but look for one that uses the "trivium" concept. There are a number of small, Christian schools in various states which do this. For math, my child started with Abeka books, and then in middle school they used Saxon. The kids took a placement test and then worked at their level and advanced through Algebra one and two. (I've heard excellent things about Singapore Math for homeschoolers as well.) Science was Bob Jones University Press. The basic 8th grade curriculum was identical to our public school's 9th grade "Integrated Physics and Chemistry" and my son got credit for the course. This was not an elite school and this was standard curriculum for every kid. Latin was started in grade school (my child went to a different Christian elementary but picked it up quickly) and he then continued in public high school, going on to national competition, until Latin was eliminated in favor of Chinese for all the immigrant kids to get easy As. Shakespeare - well, some of that had to do with the teacher, but all Christian reading texts are heavy on classical Western writing and use a higher proportion of Latin and Greek root words (see evaluation in book by Dianne Ravitch). Most decent Christian schools are a year ahead of public schools curriculum wise, and the smaller ones give you more opportunity to move your child ahead based on ability. My kid skipped two grades, but even friends of much-more average IQ two years his senior were bored once they entered our "superior" public schools.

Mitch's point about GPA is vital and people need to focus on that rather than just the SAT. Locally, there have been conflicts about which Asian was Valedictorian and the schools now have strict policies regarding which courses garner which points, and they carry out the GPA to four or five decimal points. It's ridiculous and meaningless, but it has meant that your smart White kid with a mere "A" average cannot make it into the top 10% of the class, and therefore is not guaranteed entrance to the state schools here in Texas. There's been a lot of parental anger over this, including among the Jewish cohort. The point here is it's not learning for learning's sake, it's cutthroat competition for every .01 of GPA, and due to study habits, cram schools, and parental expectations, the "winners" are always Chinese and Indian. White kids - even the truly smart ones - don't even want to compete. This is not because they are not capable, but this grade-grubbing is rightly seen as a "nerdy Asian thing" and scorned by Americans. It's a distortion of the system by outsiders for whom it was not designed.

Diversity equals chaos.
Yan Shen, go home.

Anonymous said...

But they are bad for society, because I don't like the look of them, and they are too polite(boring) for me.

It's OK for people to be interested in human beauty and to like people with extraverted, spontaneous personalities and artistic interests (I think Asians are quite at least equal to Whites in terms of artistic interest, for what it's worth).

Stormfront is far from the only place interested in these qualities, and it isn't such a big deal for people to look at Asia and find it wanting in these characteristics (best not to be rude about it though).

Of course, I think these concerns should probably not drive immigration policy.

If a black man has an IQ of 140, he is as good as any other man with an IQ of 140

Well, that would be true if people treated IQ as the sole measure of human worth, rather than just a desirable correlate. A Black man is as good as a White man if he is as good as that White man, and vice versa. Not if he has the same IQ score. That just means he has the same IQ score.

A guy with the same IQ score as Feynman or Tao, but who is not Feynman or Tao, in terms of accomplishment and genius, does not have their worth. Sad but true. Worth is worth. IQ is IQ.

Anonymous said...

Regarding talk about "gaming" the educational system:

The thing is that having a system of universities, particularly publicly subsidised ones, where people gain particular qualifications that gate their entrance into certain careers is not really an inherent good.

It's something we tolerate because gating lets us allocate resources in a way that theoretically serves the public good - either by allowing the best people to perform research or by serving as a signal to employers of good potential employees.

If "gaming the system" undermines the above goods, it's obviously bad, no matter how much people might say "well, we have to admire the cunning of the people who game the system".

I don't really see much evidence that there is Asian (or even White) "gaming" of the university system relative to Whites (or Whites relative to Blacks and Panics), or that if their is, that it undermines the above goods.

But if that is the case, it's obviously bad - studiousness and "the culture of education" are not really goods in and of themselves, and privileging people who are best able to conform to them is also not a good in and of itself. Runaway selection for "educability" that harshes other human traits like spontaneity, human warmth, actual life experience and all while not adding value - should that happen, that's bad!

There's a lot of "gaming" that has happened in our financial industry - it's obviously been bad, and even "high mathematical ability" boosters like Steve Hsu basically agree with and push this idea, that it's not the high verbal con men who've fucked our finances, but the quants and their unparseable, unrealistic, fragile and underly redundant models. Having a concern that this not happen with our educational industry is not a bad thing.

Anonymous said...

"Steve Sailer still hasn't explained why he thinks Asians studying really hard for a college entrance exam violates accepted principles of fairness(test prep as he calls it)."

Hey, what if Asians all decided to take fireman exams...

Anonymous said...

Regarding talk about "gaming" the educational system:

The thing is that having a system of universities, particularly publicly subsidised ones, where people gain particular qualifications that gate their entrance into certain careers is not really an inherent good.

It's something we tolerate because gating lets us allocate resources in a way that theoretically serves the public good - either by allowing the best people to perform research or by serving as a signal to employers of good potential employees.

If "gaming the system" undermines the above goods, it's obviously bad, no matter how much people might say "well, we have to admire the cunning of the people who game the system".

I don't really see much evidence that there is Asian (or even White) "gaming" of the university system relative to Whites (or Whites relative to Blacks and Panics), or that if their is, that it undermines the above goods.

But if that is the case, it's obviously bad - studiousness and "the culture of education" are not really goods in and of themselves, and privileging people who are best able to conform to them is also not a good in and of itself. Runaway selection for "educability" that harshes other human traits like spontaneity, human warmth, actual life experience and all while not adding value - should that happen, that's bad!

There's a lot of "gaming" that has happened in our financial industry - it's obviously been bad, and even "high mathematical ability" boosters like Steve Hsu basically agree with and push this idea, that it's not the high verbal con men who've fucked our finances, but the quants and their unparseable, unrealistic, fragile and underly redundant models. Having a concern that this not happen with our educational industry is not a bad thing.

Anonymous said...

Asians tend to be too examocentric and status-centric. Too much is rewarded in Asian society in terms who went to what school. That is important in the US too, but you can't just coast on high scores on exams and what school you went too. There are many cases of doers without fancyass credentials going very far in the America. (America's big shittiness is
pc-dentialism.)

The ironic thing about Asians is they are the most school-and-test-centric people on Earth, but their colleges are second-rate paradoxically because they are so testocentric and status-centric. In America, a student might do undergrad studies in a state college, but if he does really great work, he can get a Ph.D in an Ivy League college and move up.
I think in Asia, your future is pretty much determined in terms of what undergrad college you go to.
You're stuck for life.
(And though we think much in terms of Asian IQ, Asians seem to believe ANYONE can go to top college sheerly by effort, 'sacrifice', family support, and etc. Even the dummies are expected to make it to top college or else.)
So, if you make it to Tokyo, Seoul, or Peking university, you are really IN regardless of what you do afterwards. You matter solely cuz of your college credential. You're an important professor cuz you graduated from Tokyo university even if you never did anything special.

In the US, mediocrity is rewarded through political correctness. In Asia, mediocrity is rewarded through status correctness. Now, one might argue that one cannot be too shabbily mediocre if one passed exams to make it to elite colleges. But high scoring on exams doesn't necessarily mean one will do original or great work, and indeed most smart people don't do anything really special.

In America, at least outside clutches of PC, individuals are rewarded for originality, boldness, and groundbreakery. In Asia, not only is status more important but groundbreakery by a 'lower guy' might be seen as a kind of affront or uppityness. I mean how dare a younger guy or a guy at a lower college come up with groundbreaking stuff that a whole bunch of much-respected older guys couldn't figure out? I think there's a feeling that elite college people simply MUST BE right all the time while lesser college people should defer to elite college folks.

So, Asians doing well on exams and entering good colleges isn't itself a problem. But if all those overachievers on exams wanna go to top colleges mainly for bragging rights and social status(college as end in itself)without dreams of doing something great and original, then they are taking opportunities away from other guys who have such dream and ability.

For example, suppose there's James Smith and Fong Wong Foo. Smith is a smart guy and very curious. He reads a lot, studies hard, but sees schoolwork as a means to gain greater truth. So, he may not be obsessed about acing every exam or getting the highest possible score on SAT. Instead of obsessively focusing on SAT, he'd rather read different books, think new thoughts, and broaden his horizons. He's a candidate of someone who might achieve great stuff later. He's an A+ student but not a perfectionist A++++++++++++ student.
Fong Wong Foo, on the other hand, is limited in imagination and curiosity, but he so wanna go to good college to show how fancy and respectable he is. So, he studies only for high grades and SAT. He's a smart guy but not broad-minded or daring.

Now, who is more likely to go to top college. The Foo man. Foo man may do well in college, but is he likely to do great work? I no think it likely. But someone who might have done greater things in college will have been left out. Even so, James Smith may have the last laugh. Instead of going to undergrad Harvard, he may go to undergrad state college, but he may do really well and win Ph.D spot at Harvard whereas the Foo man might not.

Anonymous said...

It's kinda like this:
To be a truly great basketball or football player, qualities such as speed, stamina, and strength alone aren't enough.
Instead, the various abilities have to be chemistrically come together to produce something like organic athleticism. Of course, one has to be fast and strong, but that aint enough. Larry Bird, for example, wasn't the fastest or strongest but all his abilities came together to produce athletic greatness.

But suppose a college basketball team recruits players based on separate abilities in speed, strength, and stamina. So, players on judged on their sprinting time, bench press/leg press ability with weights, and stamina with jump ropes. To make the team, suppose some guys focus narrowly on speed, power, and stamina than on the
game itself.

Now, surely the most physically fit people are gonna be chosen BUT that doesn't necessarily mean they will make the best basketball players. It's possible that Carl Lewis, though superfast, would not have been naturally great at basketball. Michael Jordan was more powerfully built than most in baseball, but he failed to make it to the pros when he retired from basketball in the early 90s, and so he returned to basketball.

Same goes with genius. Though exams and SAT do choose people who are mentally able, they don't necessarily choose the sort of people whose minds will work most fluidly, interactively, and chemistrically for originality and success.
But we stick with exams cuz it's still the most objective and fairest way to assess the most number of students. And though exams can be 'gamed', they are less easier to game than more subjective or 'creative' use of criteria.

Anonymous said...

"And Mitch, shame on you for complaining about Koreans in Korea gaming the SAT, which isn't even in their native language! I'd like to see white Americans take a Chinese college entrance exam in large numbers."

Mitch may be wrong on that issue, but Asian exam culture is different from ours. There is so much pressure that I wouldn't be surprised if some kind of 'extraneous' means are employed.
I also read in some buiness news article that many Chinese and Koreans send their kids to America to study to master English the proper way cuz most English teachers in Asia tend to be second-rate. Also, it's supposed to be a kind of bragging rights to say one has been abroad and studied from the real thing--just like French restaurant chefs in America like to brag they learned from the real masters in France.

Also, there is less a culture of rule of law in China and Korea than in the US. I heard is same is true in Poland. A Polish-American friend of mine went to Poland for the medical degree, and he said cheating on exams was part of the culture; even professors knew it and tolerated it as long as it wasn't too blatant.
Japan isn't big on rule of law but it has something like code of honor rooted in samurai tradition and artisan-pride(or so I'm guessing based on movies), so Japanese are prolly less cheat-ish than Chinese and Korears.

Anonymous said...

The problem with assessing ability by tests is the matter of testophobia with some individuals. Suppose Jim and John are equally smart--or Jim is normally somewhat smarter than John--, but Jim gets nervous about exams while John is cool and relaxed about it. It could be John might score much more highly than Jim. Jim would be punished for the rest of his life cuz of his testophobia.

I aint no genius by any means, but this was true enough for me. In 8th grade, we had to take the NEDT. I didn't know what the hell it was and thought IT WAS IMPORTANT and I panicked throughout the exam. I was so nervous that my mind simply wouldn't work. I felt I was being scrutinized by the KGB. My percentile was a measly 59. Pretty pathetic.

We had to take another NEDT in highschool in sophomore yr. By that point, I didn't give a shit about school, so I took it in a leisurely manner. My percentile was 82, a big rise from 59.
82 was nothing special, but it probably better reflected my real mental ability: somwhat above average but far from genius.

But a whole bunch of Jewish kids got high 90s. That's when I knew, yep, Jews are the smartest. In contrast, many Asian kids who got good grades in school didn't score so high. Some of them scored even less than I did though they got much better school grades.
So, I knew it was more effort than natural high IQ with Asians. Jews beat all.

Anonymous said...

Yan Can Cook Shen:

"Sheila, America is my home. I love the United States more than anything else in the world."

What?!!! Even though most Americans are not high IQ?

Anonymous said...

"As is usual of the blogosphere generally and SS's blog specifically on these matters, nothing interesting or new comes through."

True, but reality is what it is. I mean should we say 'Mexicans dominate Silicon Valley' or 'Vietnamese are the up and coming thing in the NBA' just to be different?
Also, it makes sense to say the obvious over and over because it is suppressed by PC overlords. I still prefer the obvious truth over the obvious lies(that is PC).

Even so, stuff said here goes against the grain of same old same old PC lies that are the orthodoxy in mainstream culture that we hear a 100 times everyday in schools and media. It's 'you're racist, you're racist, you're racist, you're racist, and maybe sexist and homohpobic'.

Anonymous said...

"I fail to understand Steve Sailer's basic premise here, that somehow if it were the case that cramming was why Asian scores were increasing, that this would constitute "unfairly gaming" the exam."

If cram schools are just teaching knowledge and ability, that's one thing. But if their point is to look for weak spots in the exam to take advantage of, it comes somewhat close to 'gaming'. But worse, some do seem to teaching outright cheating.

Anonymous said...

"I love the United States more than anything else in the world."

"That's a lie. If you loved the United States, you would not want to see it altered beyond recognition, like something out of Bladerunner."

Actually, I wouldn't so much mind a Blade Runner future. LA in Blade Runner was kinda cool.. and didn't have too many Negroes.

rob said...

Steve Sailer said...
Gaylord Perry


Wat up wif the homophobia?

JSM said...

"it's obviously been bad, and even "high mathematical ability" boosters like Steve Hsu basically agree with and push this idea, that it's not the high verbal con men who've fucked our finances, but the quants and their unparseable, unrealistic, fragile and underly redundant models."

During the inflation of the housing bubble 1 MILLION liar's loans (euphemistically called "stated income") were made.

It couldn't have been accomplished but for the math whiz kids and their "financial engineering," their "slicing and dicing" of MBS's into tranches of, ostensibly, varying risk, so the trash could be offloaded onto gullible, (and, admittedly, greedy) pension funds.

http://www.financialsense.com/financial-sense-newshour/guest-expert/2011/09/14/william-k-black-phd/why-nobody-went-to-jail-during-the-credit-crisis

Anonymous said...

"think I've posted this before--every Indian student I've worked with has readily told me that their parents expect to arrange their marriage, and that they are "kind of hoping to talk them out of it". I'm relieved they aren't happy about it, anyway."

Too bad we don't have arranged marriages here for the white people.

Hacienda said...

Sheila,

No calculus at Trivium. Do you have the link? The other stuff is routine at any public school.

Do they teach evolution at Trivium?

Anonymous said...

"If cram schools are just teaching knowledge and ability, that's one thing. But if their point is to look for weak spots in the exam to take advantage of, it comes somewhat close to 'gaming'. But worse, some do seem to teaching outright cheating."

The point is the SAT wasn't originally designed to be studied for. Universities should either find a way to switch over to an exam that is intended to be comprehensive based on what one has studied in school or else find a way to present students with random test material that they can't know ahead of time. Perhaps we should just go to the UKs system if many people are going to insist on prepping for college entrance exams that were designed to be snapshots of potential ability rather than tests of previously studied material.

On the question of whether or not Asians are doing anything wrong: whites traditionally didn't prepare much at all for the SAT expecting that a few weeks of cramming couldn't replace years' long behavior good or bad. You can create the illusion that you are widely read, have the ability to understand and use the esoteric vocabulary on tests like the SAT and that you remember all your math b/c you were such a good student but you are still engaged in an exercise of short-term learning that will NOT stay with you after you take the SAT.

Switch to A-levels or something similar since not everyone is going to abide by the dictate not to study for a test that is supposed to approximate IQ rather than study skills/habits. If many people are going to study no matter what, it's only fair to create exams meant to test retention of previous knowledge. Heck. Use AP exams and the subject tests on the SAT for all I care just don't send that double message that the test results can't be improved by prepping when my personal experience is that they can.

I'm also interested in a renewed focus on specific abilities rather than general g. Why is it necessary to be good at both the math and verbal sections of the SAT? Most people major in something that is predicated on either their verbal IQ or their mathematical/analytical ability not both. Perhaps students who are showing a strong preference for one set of abilities over the other aren't getting into schools where they'd actually do well because, necessarily, they can't do well on all parts of the SAT.

Could it be that below a certain performance level that might indicate someone is a polymath, we're actually giving preferential treatment to less capable students?

Anonymous said...

"The ironic thing about Asians is they are the most school-and-test-centric people on Earth, but their colleges are second-rate paradoxically because they are so testocentric and status-centric. In America, a student might do undergrad studies in a state college, but if he does really great work, he can get a Ph.D in an Ivy League college and move up. "

The undergrads of their better colleges end up doing Ph.D in US(currency conversion helps a lot back home), and those who do better undergrad work at worse colleges end up doing Ph.D in their better colleges.

You see, everyone gets to move up in life, except for the group that justly deserves its comeuppance.

"But if all those overachievers on exams wanna go to top colleges mainly for bragging rights and social status(college as end in itself)without dreams of doing something great and original, then they are taking opportunities away from other guys who have such dream and ability."

And there aren't whites who do this? To gain the status of the institute that built its reputation on those who went before.
Or as noticed above in the Princeton/MIT case, white girls who beat/best white boys doing the same thing? (albeit in a different manner) and for the bragging rights of women's progress?

Anonymous said...

WN: "We should keep the foreigners out, regardless of their merits, and do our own cotton-picking and book-keeping"

Smart Roman: "We should keep the foreigners out, regardless of their merits, and do our own cotton-picking and book-keeping"

Do you see what I did there?

Anonymous said...

And if Asian culture was similarly blessed or cursed, I'm sure Asian Civ would be creative +++.

I didn't read your original comment but you seem to be saying that Europe's decentralization and diversity vs. China's centralization and homogeneity granted advantage to Europe.

If so, who forced China to be centralized? I for one think China's centralization reflects Yellow behavioral genetics, as Europe's lack reflects White behavioral genetics.

Anonymous said...

Let me put it more simply: HYPS accepts 15,000 student a year total (just making it up), of which 8,000 are white and 4000 are Asian. Out of the 8,000 whites accepted, 2000 are because their parents are large donors, former presidents, current senators, Fortune 500 CEO's, etc. The SAT scores of those 2000 kids play very little role in their admissions, cause who cares about some stupid number when your dad is a US senator.
While certainly some folks get in because of "pull" as outlined above, Felix wildly overestimates the number. There just aren't that many "Senators, fortune 500 CEOs" and folks who give multi-millions.
My data points for this are:

1. went to BigIvy as child of proles, back in the day, albeit smart proles. Saw admission process and results up close and personal.

2. Had several children. Some went to Ivies, some didn't. Saw process anew, saw their friends, competitors and classmates.

3. Have interviewed applicants for alma mater over 20 years, and also particiapted in "pool" meetings in our medium size city, where we discuss all the applciants, recommend our favorites, and then see who gets in. The number who can get in even though dumb as a box of rocks is tiny. Maybe a Kennedy or mega donor, but so few that can't possibly affect stats.

Where pull operates, and even then not for many, is from the same pool that would be near the bottom of the admits -- so replacing a "pull" admit with a "deserving" non-pull, would again affect the stats barely at all.

ANd for those who think that all or many "alumini" admits must be dummies, a study done in connection with suit by Asians against harvard admissions showed that alum admits were within 25 SAT points (of 1600, I think -- possibly of 800) of racially comparable non-alum admits.

Where someone might like to do a rigorous study would be of PBK memebrs. They do it strictly on grades -- as shown by the infinitesimal NAM representation at BigIVy -- if anyone noticed that, they could make it a "scandal" but no one has yet!

Anonymous said...

This place is becoming a lot like stormfront.

Indeed; the trolls just don't #*@!ing read & comprehend. Or don't respond as if they do.

Ok, so asians do better in [our] school,

- While not providing any school on their turf for our kids to go to in similar numbers or proportions -

do much better in mathematics,

- While keeping us out of any place of theirs where our maths skills (or our fortes, if you really want to reciprocate) would be compared to theirs -

become scientists, inventors, and doctors at a higher rate than white people.

- Again, on our turf, while locking us out of their turf which, even were they to allow us access is shit compared to ours -

But they are bad for society, because I don't like the look of them, and they are too polite(boring) for me.

- Meanwhile, Yellows don't even have to bitch about White faces or habits because they lock them out from their territory altogether -

I mean, really, sir, a hearty fuck you to you.

Whites excel Yellows at media & entertainment, oratory, politics, law, literature, acting, art, etc. Note I didn't even mention innovation per se. So how about we let Whites into all those areas in China? See who wins the "fair competition," right? See how the Chinese like having their politics, media, culture, arts and law dominated by Whites.

What's ours is not yours, sir, it is ours.

The temerity! The dishonesty! It is to laff.

RKU said...

This is hilarious. All the effort to prove that black people cannot catch up to whites no matter how hard they try in HBD circles comes unraveled when it comes to Asians...Apparently the reason that blacks dominate the NFl combine is because whites and especially Asians do not spend enough time in the gym.

Well, reading through much of this long thread, I get the eerie sense that most of the local WN-types seem to be fierce critics of "Rushton's Rule."

...Which seems a bit odd to me, given that Phil Rushton is (arguably) the world's most prominent WN academic intellectual...

Anonymous said...

I've discovered a fairly good compromise here. Since Mitch seems to be complaining about the new SAT math section and since Steve Sailer also seems to be suggesting that the new SAT is somehow less g-loaded and more amenable to test prep, I suggest we replace the SAT math section with the extremely g-loaded Raven's Progressive Matrices. That should uh help close the Asian-white gap...

Anonymous said...

I'd like to see white Americans take a Chinese college entrance exam in large numbers.

Then make China vastly more attractive to Americans than America is, and make the Chinese as accepting of outsiders and immigration as Americans are, and you will.

Anonymous said...

"This is hilarious. All the effort to prove that black people cannot catch up to whites no matter how hard they try in HBD circles comes unraveled when it comes to Asians"

No, the issue is not 'whites catching up to Asians' but 'Asians improving their scores more than expected'. Even so, the difference between whites and Asians is pretty narrow, so I would say it's more a matter of Asian effort than natural ability.

Anonymous said...

I think we should beware of crutch-ism.

White crutch argument goes like this: Asians may try harder(cuz they are conformist robots)but we got the creativity. Maybe so, but creativity without effort and concentration don't amount to much. So effort + creativity is great, but no one should use 'creativity' as a crutch for being lazy or self-indulgent. There are too many young people who act lazy, stupid, and weird but excuse their lame behavior as 'creative'. So, creativity should be used as a tool, not as a crutch.

Asians seem to suffer from the crutchism of effort. They seem to think if you get good grades and test well and go to good school, that is the end-all of life. That too is a kind of crutch. So, if some Asian kid goes to elite school, he thinks he's better simply because he's part of the exclusive community EVEN IF he himself does nothing great in life.

So, in this case, both sides need to learn. Whites need more effort(to really put their creativity to good use), and Asians need more freedom to more inventive with their effort.

Anonymous said...

WE WANT NAMIE AMURO!!

Anonymous said...

"Nobody has a good way of measuring people's creative potential, and people's creative achievement..."

Yes, you can measure a peoples creative potential. Most things that are considered to be "unmeasurable" just need to be broken down into their components first.

Here's one way you could measure creativity in U.S. for instance

#of patents by whites/overall # of patents. Say this is 80% and the average age is 50. If the population proportion for white 50 year olds is 75% then on average whites are more creative then others.

I think there are plenty of ways to measure creativity and in all of them whites far exceed other cultures. It's self-evident.

Anonymous said...

Also, it makes sense to say the obvious over and over because it is suppressed by PC overlords. I still prefer the obvious truth over the obvious lies(that is PC).

Absolutely. The poor sap who believes everything the authorities tell him might've just wandered in here today, after all.

RKU, I find Rushton's scale very useful (and predictive) and try to incorporate it throughout my HBD perspectives.

Anonymous said...

"I suggest we replace the SAT math section with the extremely g-loaded Raven's Progressive Matrices. That should uh help close the Asian-white gap..."

Won't that widen the gap? My understanding is that the 3-5 point advantage attained by Asians over whites on IQ tests is almost entirely due to visuospatial inventories. I would recommend supplementing the SAT with the Miller Analogies Test. I'd also like to know how Asians perform relative to whites on the ASVAB which is probably a more accurate measure of real world smarts than the SAT or ACT.

Anonymous said...

Actually, though some white folks panic about them super-striving yellers, I say let them remain exam-centric cuz they'll never become dominant that way.

What we really need to fear are the following:

1. Asians learning to focus more on IQ and eugenics than effort and examics. Why did Japan lose WWII--among other things? They thought purity of spirit and super-effort could overcome anything. Why did US invent the bomb? We had the genius Jews!!! So, the real secret to greatness is IQ, but Asians haven't figured it out yet. They have a system that doesn't so much reward high IQ as high anxiety. Of course, Asian exam system favors the smarties over the dammies, but the pressures and narrow focus ill serve smart people.
The whole test-system, instesad of nurturing and encouraging free-thinking and originality, turns even smart people into rote-memory drones. Also, rote-ism also favors rotos over ratos. Rotos are people who are superb with memory but not necessarily good thinkers. Ratos would be people who are good with rational stuff, problem solving. I heard Stalin and Mao had elephant-like memory, but they were not exactly logical when it came to stuff like economics. In the movie PAPER CHASE, one guy has a photographic memory, but he's not good at putting all the data together to make a good argument. As long as Asia favors rotos over ratos, they cannot win.
Also, though effort is necessary(no shit), MORE EFFORT DOESN'T NECESSARILY MEAN MORE ACHIEVEMENT. Everyone has his limits and reaches a plateau. It's like no matter how hard you train, you're not gonna be Usain Bolt or the Iranian gold medal weightlifter. Understanding of biology and genetics makes people realize this, but Asians haven't figured it out yet. They are still stuck in the Confucian cultural rut of 'more effort make better'. They are wasting a lot of mental and social energy inefficiently.

Anonymous said...

The new Asian business elite rising in America. What would we due without their incredible talents? Why, we'd be jettisoned back to the stone age, of course.

http://www.nytimes.com/2011/09/17/nyregion/couple-accused-of-stealing-food-money-from-red-apple-preschools.html?_r=2&partner=MYWAY&ei=5065

Anonymous said...

"I would recommend supplementing the SAT with the Miller Analogies Test."


I dunno. That is like the world's easiest test.

Anonymous said...

Roots of Korean obsession?

I guess N. Korea doesn't count in this equation.

Disappointed Asian said...

@Zylonet

I did not mean a "Team White" or "Team Asian". I'm very aware that no matter how narrow an in-group becomes, there will always be a reason to make it smaller by highlighting other differences.

My original point was that I thought HBD's emphasis on intelligence transcends race, so it'd be more of "Team Smarts" and "Team Idiots".

Since whites and Asians have high average IQs and share similar values, it's more likely that both groups would get along better (relative to NAMs).

But instead I am getting the feeling here that HBD is being used to advance a pro-white stance rather than a pro-intelligence one. This is done by coming up with rationals as to why white people are better and some of it is pretty ridiculous. For example, being "creative" and "artistic" is being trumped up as a form of intellect. I didn't know the theory of multiple intelligences carried weight in the HBD sphere.

To me, this is foolish and for obvious reasons, something I can not stand behind.

In my experience, I feel a lot more comfortable around people who are closer to my intelligence level regardless of race. We've all been there where we are surrounded by people who are dimmer than us and it creates tension and awkwardness. I'm not a genius or anything, but I think you can all understand from your own experiences.

Matt said...

"I suggest we replace the SAT math section with the extremely g-loaded Raven's Progressive Matrices. That should uh help close the Asian-white gap..."

This is basically facetious, but (because I want to talk about it anyway)....

Firstly, Raven's can be studied for as well - my understanding from the literature is that it's highly fluid g loaded (even more than being g loaded), because it's a bizarre set of tasks that don't really have any application in real life, so because it's novel, people have to solve them using their own ability to reason.

Of course, you can still only learn strategies for Raven's not actually memorise the solutions, so there's still a strong reasoning component, but if you made the Raven's the golden measure rather than some weird thing that noone has learnt to do, then you would get people studying for the test, successfully.

Secondly, if you make Raven's the golden measure, you also lose the ability to distinguish math and verbal ability, which is probably more interesting than their general factor score, for specific majors.

However, you can compensate for this. Interestingly, the reason why Raven's is so g-correlated is because the items are a mix of those best solvable by verbal-analytic and visuo-spatial strategies (and the "harder" and easier items aren't necessarily visuo-spatial). There's a paper by DeShon et al that shows this and subsequent confirmatory papers confirm this - http://tinyurl.com/5wnkhfb. There's no "pure reasoning" - it just happens that Raven's is relatively closer to loading on the all the skills a typical test battery loads on compared to any other single test, so it correlates better with the "general factor" that would emerge from that battery.

As an aside, this would probably be interesting from a HBD-IQ perspective as it provides a language neutral way of testing "verbal IQ", which would let us test the Asian verbal IQ to all the doubters who say there is no relative defecit relative to "spatial IQ" or absolute defecit relative to Whites and thing it's just some 2L non-native language effect (I think the first is relatively likely or the second is relatively unlikely).

It would also show us where the Asian advantage on Raven's comes from - my guess would be Asian advantage would only exist on visuo-spatial items, or at least most strongly, as in people with High Functioning Autism (and autism in general). It would also help us work out whether the Asian Raven's advantage relative to full scale IQ scores is mainly due to a slight bias to include more "visual-spatial" problems in the Raven's relative than exist in the whole full scale IQ battery, or whether this isn't relevant.

Anonymous said...

...Which seems a bit odd to me, given that Phil Rushton is (arguably) the world's most prominent WN academic intellectual...

I think this may be more a case of "Phil Rushton is (arguably) the world's most prominent (arguably) WN academic intellectual..." I don't really such much WN from the guy other than opposing immigration to the USA, but he only does so for non-East Asian countries (unless I'm wrong) and is married to some Asian chick.

Ivy League Bastard said...

The postings by Mitch and RKU contain all the content in this 150+ comment discussion.

A few footnotes to their contributions.

> from say 2000-2005 the Asian improvement
>in SD units relative to whites on the math
>and verbal sections of the SAT was about the
> same as the improvement from 2005-2010.
> This seems to cast some doubt on Mitch's theory

No, it is further evidence for the "test prep" thesis. There will at some point be a leveling off, but the white/Asian gap should continue increasing as part of a trend that started much earlier.

As the immigrant communities get larger and Asian suburban enclaves proliferate, there is (per capita) a parallel growth in the supply and demand for afterschool instruction, and the money to pay for it. Extracurricular academic offerings tend to expand, start earlier and earlier in childhood, and are better coordinated with school curricula and tests.

As a result, the Chinese and Korean college prep industry has matured well beyond the stage of cram schools in Flushing. White, English fluent tutors are added to the teaching staff, in addition to Asian immigrants. The overproduction of underqualified (in real terms) Asian Ivy League grads has a natural relief valve in these places, where their test scores are meaningful and a ticket to coaching the next student generation.

I think there is also a demographic shift in the recent SAT-takers, from ESL immigrants to a higher proportion of native speakers, which raises the verbal scores.

Test prep itself has become more streamlined, due to the internet, competition between the multitude of tutors (especially in Asian communities), more SATs being revealed over time, and the test becoming a more coachable conscientiousness test as Mitch outlined.

No study, least of all a self-interested study by ETS, has shown that gains from test prep are limited to a small number of points or that prep is ineffective. Those studies at best describe the average case of short term, low intensity preparation in a population selected for *not* having previously used test prep.

The preparation process, for all students but especially as practiced in Asian immigrant communities, takes many years and can be influenced toward higher scores on an almost daily basis. The environmental influence is enormous if one chooses to apply it, and Asians do so choose. "Cram schools" is a misnomer if it suggests that short-term coaching is responsible for the testing gains. What happens is not a 200 point jump in several months (though I guess this is sometimes possible) but a sum of small creeping increments cumulated over a decade, with larger test-specific prep appearing at a late stage to exacerbate the disparities between groups.

One can't fault the Asian parents for pursuing these strategies, but the end result is a large inflation of scores beyond their natural ability-based levels. That's true whether or not the levels are the same as whites. And as Mitch wrote, this applies even more so to grades, which are more under a student's (and parents') control than test scores.

Anonymous said...

"lol if you wanna say who Zuckerberg purportedly stole the idea of Facebook from at least mention the Winklevoss' partner (i.e. co-founder of HarvardConnection/ConnectU) Divya Narendra.

But that would dilute your message wouldn't it. "

wait, who was that in that movie? Divya is a common girl's name, so I was trying to recall a swarthy babe...the gall of portraying him with a white actor!

The Anti-Gnostic said...

I suggest white Americans who feel threatened should be ready to adapt traits that can give them the best advantage as well. Otherwise, you know, natural selection will simply run its course.

Perhaps. OTOH, Orientals are aborting a lot of daughters and the ones they carry to term are out-marrying at higher rates than their brother Orientals.

I don't have any particular beef with Orientals. I certainly prefer them to the Meso-American/African apocalypse that awaits us. But I question their ability to grasp the importance of limited government.

I would love to be proved wrong, but my opinion over years of contact with different ethnicities is that lex rex is pretty much an Anglo-Saxon attitude. And when it's gone, it's gone.

Anonymous said...

"I would recommend supplementing the SAT with the Miller Analogies Test. I'd also like to know how Asians perform relative to whites on the ASVAB which is probably a more accurate measure of real world smarts than the SAT or ACT."

Your IQ should be evident in your performance. If we had national subject tests, each high school would have to tailor their curriculum so their students were learning what was on the standardized subject test. The smarter the student, the likelier he would get a test prep book anyway.

Grade inflation could be cancelled out somewhat by requiring other documentation of ability, be it a portfolio of your coursework or success in an extracurricular activity closely related.

I'm ever more interested in using achievement tests rather than IQ tests to determine college entry. It looks to me as if ETS has more found a way to study IQ by getting huge numbers of people to take the SAT than created a quick, simple method to determine who has the minimum qualifications to attend your school.

I suspect the attitude persists that you're some kind of chump if you love and do well in a subject like English or history yet don't have the SAT/IQ score that defines you as part of an intellectual elite. IMO, it's positively demotivating. Much better for someone with a passion yet maybe a more average IQ to get into an elite institution than for an unmotivated, underachiever to get a slot largely because of IQ.

Anonymous said...

Where did all the Japanese-American brains go? Here's one... or are Jews losing interest in chess in favor of computers?

Anonymous said...

"Nakamura was born in Hirakata, Osaka Prefecture, Japan, to a Japanese father and an American mother."

Maybe his mother is Jewish.

Anonymous said...

Actually the huge mountain of evidence favors superior white intelligence: It's the historical record. Whites have produced wildly disproportionate intellectual contributions vs. Asians in almost every field of human endeavor over the past, what, 2000 years.

Like another poster said, its actually about 500 years, and the rate of contribution has been tapering off since the late 19th C., at least according to Charles Murray in On Human Achievement.

If Spengler is on to something, civilizations have a cycle that feeds and them reduces human accomplishment. In other words, in another hundred years or so, whites will have become much more Asian.

Anonymous said...

PSAT advice:

I generally agree with JSM, but do have your son take a few practice tests. It builds confidence, which makes a difference for a lot of kids at the margin.

-osvaldo m.

Anonymous said...

"The environmental influence is enormous if one chooses to apply it, and Asians do so choose. "Cram schools" is a misnomer if it suggests that short-term coaching is responsible for the testing gains. What happens is not a 200 point jump in several months (though I guess this is sometimes possible) but a sum of small creeping increments cumulated over a decade, with larger test-specific prep appearing at a late stage to exacerbate the disparities between groups."

blah blah Black-White difference is genetic blah blah no environmental effect from differences between what black and white kids experience blah blah But Asian kids get 200 points from parental investment over a decade ...

The Anti-Gnostic said...

blah blah Black-White difference is genetic blah blah no environmental effect from differences between what black and white kids experience blah blah But Asian kids get 200 points from parental investment over a decade ...

We will just have to go the Full Bullock on every black kid in America. No Child Left Behind!

Anonymous said...

Asians are smarter since they're overrepresented in the highest science and mathematics competitions and they have higher g than whites as early as primary school and do as well even when they are adopted by white parents. China has dominated the world for 18 of the last 20 centuries technologically and economically and Asians have dominated the world technologically for roughly 40,000 years and Asians were more peaceful than Whites.

http://www.youtube.com/watch?v=-t3QSGRN1-U

@1:28:30

THE CHINESE WEREN'T GREEDY ENOUGH

Those were the words of a White professor. White happen to have industrialized before Asians because of liberalism and the fact they were more psychopathic (more serial killers, more megalomaniac leaders) than Asians.

Anonymous said...

Those were the words of a White professor. White happen to have industrialized before Asians because of liberalism and the fact they were more psychopathic (more serial killers, more megalomaniac leaders) than Asians.

Yellows only industrialized when they did because Whites pioneered industrialization. God only knows how long it would have taken Yellows to do it on their own.

You imply that liberalism was incidental; it wasn't. I like the "not being ants=psychopathic" though, nice touch.

Anonymous said...

China has dominated the world for 18 of the last 20 centuries technologically and economically

Btw, whatchu smokin'?

torporify said...

"Yellows." I love it.
btw, no European disaster ever produced cannibalism (of the ritual and perhaps medicinal sort) that occurred in China during the "Cultural Revolution" of the late 60s. Pretty well covered by a Chinese journalist who just had to "go there" and find out. Yes, it really happened; not occassionally but frequently.
Also, the behavior of the Nazis during WWII had nothing on that of the Japanese in China (especially.) All they needed was a bit more time, and I have no doubt they would have overtaken the Germans in that area just as efficiently as they took over the auto industry. The tortures recounted in Cambodia are so imaginative (one involving a sort of clockwork arrangement that decapitated the prisoner children one by one) that I have no doubt of Yellow creativity equalling White.
I am not sure why "serial killers" keep getting brought up as some sort of indictment of whites. Reminds me of how suicide was thought to be higher in Scandinavian countries because they were so unhappy. Turns out they were happy enough, and their suicide rates were not much higher. They were just more honest and better organized about reporting it, not because it is really higher. The crime rates of the White countries (until they insanely import a lot of blacks and Muslims) is so low that I think serial killers are among the last of their concerns.

Anonymous said...

ZZ said...
The informal quota system used at elite private schools is a decent compromise between two extremes (excluding foreign HS students for simplicity):

SAT based admissions: 65% white, 34% asian, 1% NAM

Population based admissions: 65% white, 32% NAM, 3% asian

Actual "soft quota compromise" numbers at HYPS: 65% white, 20% asian, 15% NAM."




Very interesting ZZ.

Now could you redo that whole thing using Christian, Jewish and other, instead of Asian, white and NAM?

Thanks

Anonymous said...

So, will Asians get all excited about remedying the Asian-Black gap?

Do celebrity Asians want to adopt black kids?

Tiger Asian moms with black kids out of the foster care system, anyone?

Can you even imagine such things?

Matt said...

This is a dead thread now, but...

For people who focus on this change as driven by demographic change in the pool of Asian immigrants, note wikipedia:

http://tinyurl.com/3bnub2b

Converted to percentage terms as follows:

http://s000.tinyupload.com/?file_id=00944929555963856207

Now, if we look at this in percentage terms - the change in percentage composition for all Asian subgroups as a percent of the total is less that 1% for each group, except Japanese Americans who drop by a couple percentage points, and Indian and Other Asian Americans, who increase by 2% of the total each.

I don't think that's grounds for this relating to an increase in "Chinese share" or "North East Asian share" amongst Asian Americans. There's actually a decrease in North East Asian share (Chinese, Japanese, Korean) from 42% to 38% (and every subgroup decreases in share), a slight decrease in South East Asian share (measured by Filipinos and Vietnamese) from 29% to 28% and a larger increase in the Indian American share from 16% to 19% and in the "Other Asian American" share from 12.5% to 15%. Demographic change can't explain increases in performance - Indians are good, but not that good.